+7 495 120-13-73 | 8 800 500-97-74

(для регионов бесплатно)

Содержание

Закон Кулона, конденсатор, сила тока, закон Ома, закон Джоуля – Ленца | ЕГЭ по физике

Закон Кулона

Закон Кулона — это один из основных законов электростатики. Он определяет величину и направление силы взаимодействия между двумя неподвижными точечными зарядами.

Под точечным зарядом понимают заряженное тело, размер которого много меньше расстояния его возможного воздействия на другие тела. В таком случае ни форма, ни размеры заряженных тел не влияют практически на взаимодействие между ними.

Закон Кулона экспериментально впервые был доказан приблизительно в 1773 г. Кавендишем, который использовал для этого сферический конденсатор. Он показал, что внутри заряженной сферы электрическое поле отсутствует. Это означало, что сила электростатического взаимодействия меняется обратно пропорционально квадрату расстояния, однако результаты Кавендиша не были опубликованы.

В 1785 г. закон был установлен Ш. О. Кулоном с помощью специальных крутильных весов. 2}$

где $|q_1|$ и $|q_2|$ — модули зарядов; $r$ — расстояние между ними; $k$ — коэффициент пропорциональности, зависящий от выбора системы единиц. Сила взаимодействия направлена по прямой, соединяющей заряды, причем одноименные заряды отталкиваются, а разноименные — притягиваются.

Сила взаимодействия между зарядами зависит также от среды между заряженными телами.

В воздухе сила взаимодействия почти не отличается от таковой в вакууме. Закон Кулона выражает взаимодействие зарядов в вакууме.

Кулон — единица электрического заряда. Кулон (Кл) — единица СИ количества электричества (электрического заряда). Она является производной единицей и определяется через единицу силы тока 1 ампер (А), которая входит в число основных единиц СИ.

За единицу электрического заряда принимают заряд, проходящий через поперечное сечение проводника при силе тока $1$А за $1$с.

То есть $1$ Кл$= 1А·с$.

Заряд в $1$ Кл очень велик. Сила взаимодействия двух точечных зарядов по $1$ Кл каждый, расположенных на расстоянии $1$ км друг от друга, чуть меньше силы, с которой земной шар притягивает груз массой $1$ т. 2$ — электрическая постоянная.

Электрическая емкость конденсатора

Электроемкость

Электроемкостью проводника $С$ называют численную величину заряда, которую нужно сообщить проводнику, чтобы изменить его потенциал на единицу:

$C={q}/{φ}$

Емкость характеризует способность проводника накапливать заряд. Она зависит от формы проводника, его линейных размеров и свойств среды, окружающей проводник.

Единицей емкости в СИ является фарада ($Ф$) — емкость проводника, в котором изменение заряда на $1$ кулон меняет его потенциал на $1$ вольт.

Электрический конденсатор

Электрический конденсатор (от лат. condensare, буквально сгущать, уплотнять) — устройство, предназначенное для получения электрической емкости заданной величины, способное накапливать и отдавать (перераспределять) электрические заряды.

Конденсатор — это система из двух или нескольких равномерно заряженных проводников с равными по величине зарядами, разделенных слоем диэлектрика.

2}/{2}$

где $ε$ — диэлектрическая проницаемость среды, $ε_0$ — электрическая постоянная.

Сила тока

Электрическим током называется упорядоченное (направленное) движение заряженных частиц.

Сила электрического тока — это величина ($I$), характеризующая упорядоченное движение электрических зарядов и численно равная количеству заряда $∆q$, протекающего через определенную поверхность $S$ (поперечное сечение проводника) за единицу времени:

$I={∆q}/{∆t}$

Итак, чтобы найти силу тока $I$, надо электрический заряд $∆q$, прошедший через поперечное сечение проводника за время $∆t$, разделить на это время.

Сила тока зависит от заряда, переносимого каждой частицей, скорости их направленного движения и площади поперечного сечения проводника.

Рассмотрим проводник с площадью поперечного сечения $S$. Заряд каждой частицы $q_0$. В объеме проводника, ограниченном сечениями $1$ и $2$, содержится $nS∆l$ частиц, где $n$ — концентрация частиц. 2$, дает весьма незначительную величину — $∼0.1$ мм/с.

Закон Ома для участка цепи

Сила тока на участке цепи равна отношению напряжения на этом участке к его сопротивлению.

Закон Ома выражает связь между тремя величинами, характеризующими протекание электрического тока в цепи: силой тока $I$, напряжением $U$ и сопротивлением $R$.

Закон этот был установлен в 1827 г. немецким ученым Г. Омом и поэтому носит его имя. В приведенной формулировке он называется также законом Ома для участка цепи. Математически закон Ома записывается в виде следующей формулы:

$I={U}/{R}$

Зависимость силы тока от приложенной разности потенциалов на концах проводника называется вольт-амперной характеристикой (ВАХ) проводника.

Для любого проводника (твердого, жидкого или газообразного) существует своя ВАХ. Наиболее простой вид имеет вольт-амперная характеристика металлических проводников, заданная законом Ома $I={U}/{R}$, и растворов электролитов.

Знание ВАХ играет большую роль при изучении тока.

Закон Ома — это основа всей электротехники. Из закона Ома $I={U}/{R}$ следует:

  1. сила тока на участке цепи с постоянным сопротивлением пропорциональна напряжению на концах участка;
  2. сила тока на участке цепи с неизменным напряжением обратно пропорциональна сопротивлению.

Эти зависимости легко проверить экспериментально. Полученные с использованием схемы, графики зависимости силы тока от напряжения при постоянном сопротивлении и силы тока от сопротивления представлены на рисунке. В первом случае использован источник тока с регулируемым выходным напряжением и постоянное сопротивление $R$, во втором — аккумулятор и переменное сопротивление (магазин сопротивлений).

Электрическое сопротивление

Электрическое сопротивление — это физическая величина, характеризующая противодействие проводника или электрической цепи электрическому току.

Электрическое сопротивление определяется как коэффициент пропорциональности $R$ между напряжением $U$ и силой постоянного тока $I$ в законе Ома для участка цепи.

Единица сопротивления называется омом (Ом) в честь немецкого ученого Г. Ома, который ввел это понятие в физику. Один ом ($1$ Ом) — это сопротивление такого проводника, в котором при напряжении $1$ В сила тока равна $1$ А.

Удельное сопротивление

Сопротивление однородного проводника постоянного сечения зависит от материла проводника, его длины $l$ и поперечного сечения $S$ и может быть определено по формуле:

$R=ρ{l}/{S}$

где $ρ$ — удельное сопротивление вещества, из которого изготовлен проводник.

Удельное сопротивление вещества — это физическая величина, показывающая, каким сопротивлением обладает изготовленный из этого вещества проводник единичной длины и единичной площади поперечного сечения.

Из формулы $R=ρ{l}/{S}$ следует, что

$ρ={RS}/{l}$

Величина, обратная $ρ$, называется удельной проводимостью $σ$:

$σ={1}/{ρ}$

Так как в СИ единицей сопротивления является $1$ Ом, единицей площади $1м^2$, а единицей длины $1$ м, то единицей удельного сопротивления в СИ будет $1$ Ом$·м^2$/м, или $1$ Ом$·$м. {-1}$. Для растворов электролитов $α

Зависимость сопротивления проводника от температуры используется в термометрах сопротивления.

Параллельное и последовательное соединение проводников

Для параллельного соединения проводников справедливы следующие соотношения:

1) электрический ток, поступающий в точку $А$ разветвления проводников (она называется также узлом), равен сумме токов в каждом из элементов цепи:

$I=I_1+I_2;$

2) напряжение $U$ на концах проводников, соединенных параллельно, одно и то же:

$U=U_1=U_2;$

3) при параллельном соединении проводников складываются их обратные сопротивления:

${1}/{R}={1}/{R_1}+{1}/{R_2}, R={R_1·R_2}/{R_1+R_2};$

4) сила тока и сопротивление в проводниках связаны соотношением:

${I_1}/{I_2}={R_2}/{R_1}$

Для последовательного соединения проводников в цепи справедливы следующие соотношения:

1) для общего тока $I$:

$I=I_1=I_2,$

где $I_1$ и $I_2$ — ток в проводниках $1$ и $2$ соответственно; т. е. при последовательном соединении проводников сила тока на отдельных участках цепи одинакова;

2) общее напряжение $U$ на концах всего рассматриваемого участка равно сумме напряжений на отдельных его участках:

$U=U_1+U_2;$

3) полное сопротивление $R$ всего участка цепи равно сумме последовательно соединенных сопротивлений:

$R=R_1+R_2;$

4) также справедливо соотношение:

${U_1}/{U_2}={R_1}/{R_2}$

Работа электрического тока. Закон Джоуля-Ленца

Работа, совершаемая током, проходящим по некоторому участку цепи, согласно ($U=φ_1-φ_2={A}/{q}$) равна:

$A=qU$

где $А$ — работа тока; $q$ — электрический заряд, прошедший за данное время через рассматриваемый участок цепи. Подставляя в последнее равенство формулу $q=It$, получаем:

$A=IUt$

Работа электрического тока на участке цепи равна произведению напряжения на концах этого участка на силу тока и на время, в течение которого совершалась работа.

Закон Джоуля-Ленца

Закон Джоуля — Ленца гласит: количество теплоты, выделяемое в проводнике на участке электрической цепи с сопротивлением $R$ при протекании по нему постоянного тока $I$ в течение времени $t$ равно произведению квадрата тока на сопротивление и время:

$Q=I^2Rt$

Закон был установлен в 1841 г. 2}/{R}$

Из соотношения для ЭДС легко получить мощность источника тока:

$P_ε=εI$

В СИ работу выражают в джоулях (Дж), мощность — в ваттах (Вт), а время -в секундах (с). При этом

$1$Вт$=1$Дж/с, $1$Дж$=1$Вт$·$с.

Рассчитаем наибольшую допустимую мощность потребителей электроэнергии, которые могут одновременно работать в квартире. Так как в жилых зданиях сила тока в проводке не должна превышать $I=10$А, то при напряжении $U=220$В соответствующая электрическая мощность оказывается равной:

$Р=10А·220В=2200Вт=2.2кВт.$

Одновременное включение в сеть приборов с большей суммарной мощностью приведет к увеличению силы тока, и потому недопустимо.

В быту работу тока (или израсходованную на совершение этой работы электроэнергию) измеряют с помощью специального прибора, называемого электрическим счетчиком (счетчиком электроэнергии). При прохождении тока через этот счетчик внутри его начинает вращаться легкий алюминиевый диск. Скорость его вращения прямо пропорциональна силе тока и напряжению. Поэтому по числу оборотов, сделанных им за данное время, можно судить о работе, совершенной током за это время. Работа тока при этом выражается обычно в киловатт-часах ($кВт·ч$).

$1кВт·ч$ — это работа, совершаемая электрическим током мощностью $1кВт$ в течение $1ч$. Так как $1кВт=1000Вт$, а $1ч=3600с$, то $1кВт·ч=1000Вт·3600с=3600000 Дж$.

Работа и мощность тока — урок. Физика, 8 класс.

Напряжение показывает, какую работу совершает электрическое поле при перемещении единичного положительного заряда из одной точки в другую.

U=Aq, где \(U\) – напряжение, \(А\) – работа тока, \(q\) – электрический заряд.

Таким образом, говоря другими словами,

напряжение на концах участка цепи численно равно работе, которая совершается при прохождении по этому участку электрического заряда в 1 Кл.

При прохождении по этому же участку электрического заряда, равного не \(1\) Кл, а, например, \(10\) Кл, совершённая работа будет в \(10\) раз больше.
Это означает, чтобы определить работу электрического тока на каком-либо участке цепи, надо напряжение на концах этого участка цепи умножить на электрический заряд, прошедший по нему: A=U⋅q,
Для выражения любой из величин можно использовать приведенные ниже рисунки.
Электрический заряд, прошедший по участку цепи, можно определить, измерив силу тока и время его прохождения: q=I⋅t. Используя это соотношение и подставляя его в формулу A=U⋅q, получим формулу для нахождения работы электрического тока: A=U⋅I⋅t.

Работа электрического тока на участке цепи равна произведению напряжения на концах этого участка на силу тока и на время, в течение которого совершалась работа.

Чтобы выразить любую из величин из данной формулы, можно воспользоваться рисунком


Как известно, работу измеряют в джоулях, напряжение — в вольтах, силу тока — в амперах и время в секундах.

 

Тогда  1 джоуль = 1 вольт · 1 ампер · 1 секунду, или 1 Дж = 1 В · А ·С.


Из выше сказанного следует, что для измерения работы электрического тока нужны вольтметр, амперметр и часы. Например, для определения работы, которую совершает электрический ток, проходя по спирали лампы накаливания, необходимо собрать цепь, изображенную на рисунке. Вольтметром измеряется напряжение на лампе, амперметром – сила тока в ней. А при помощи часов (секундомера) засекается время горения лампы.

 

Например,

 

I = 1,2 АU = 5 Вt = 1,5 мин = 90 сА = U⋅I⋅t = 5⋅1,2⋅90 = 540 (Дж) 

 

Обрати внимание!

Работа чаще всего выражается в килоджоулях или мегаджоулях.

\(1\) кДж = 1000 Дж или \(1\) Дж = \(0,001\) кДж;
\(1\) М Дж = 1000000 Дж или \(1\) Дж = \(0,000001\) МДж.

На практике работу электрического тока

измеряют специальными приборами — счётчиками. Счётчики электроэнергии можно видеть в каждом доме.

 

 

Из курса физики известно, что мощность численно равна работе, совершённой в единицу времени N = Аt. Следовательно, чтобы найти мощность электрического тока, надо его работу  A=U⋅I⋅t разделить на время.

В отличие от механической мощности мощность тока обозначают буквой \(Р\).

P=At=U⋅I⋅tt=U⋅I. Отсюда следует, что

мощность электрического тока равна произведению напряжения на силу тока: P=U⋅I.

Из этой формулы можно определить и другие физические величины.
Для удобства можно использовать приведенные ниже рисунки.

 

За единицу мощности принят ватт; \(1\) Вт = \(1\) Дж/с.

 

Из формулы P=U⋅I следует, что


\(1\) ватт =\(1\) вольт х \(1\) ампер, или \(1\) Вт = \(1\) В ∙ А.

Обрати внимание!

Используют также единицы мощности, кратные ватту: гектоватт (гВт), киловатт (кВт), мегаватт (МВт).
\(1\) гВт = \(100\) Вт или \(1\) Вт = \(0,01\) гВт;
\(1\) кВт = \(1000\) Вт или \(1\) Вт = \(0,001\) кВт;
\(1\) МВт = \(1 000 000\) Вт или \(1\) Вт = \(0,000001\) МВт.

Измерить мощность электрического тока можно с помощью вольтметра и амперметра.

 

 

Чтобы вычислить искомую мощность, необходимо напряжение умножить на силу тока. Значение силы тока и напряжение определяют по показаниям приборов.

 

I = 1,2 АU = 5 ВP = U⋅I = 5⋅1,2 = 6 (Вт)

 

Существуют специальные приборы — ваттметры, которые непосредственно измеряют мощность электрического тока в цепи. Они бывают аналоговые и цифровые. В зависимости от сферы применения у них различаются пределы измерения.

 

аналоговый ваттметр

аналоговый ваттметр

аналоговый ваттметр

цифровой ваттметр

 

Подключим к цепи по очереди две лампочки накаливания, сначала одну, а затем другую и измерим силу тока в каждой из них. Она будет разной.

 

 

 

Сила тока в лампочке мощностью \(25\) ватт будет составлять \(0.1\) А. Лампочка мощностью \(100\) ватт потребляет ток в четыре раза больше — \(0.4\) А. Напряжение в этом эксперименте неизменно и равно \(220\) В. Легко можно заметить, что лампочка в 100 ватт светится гораздо ярче, чем \(25\)-ваттовая лампочка. Это происходит от того, что ее мощность больше. Лампочка, мощность которой в \(4\) раза больше, потребляет в \(4\) раза больше тока. Это показывает, что

 

Обрати внимание!

мощность прямо пропорциональна силе тока.

Что произойдет, если одну и ту же лампочку подсоединить к источникам различного напряжения? В данном случае используется напряжение \(110\) и \(220\) В.


  Можно заметить, что при большем напряжении лампочка светится ярче, значит, в этом случае ее мощность будет больше. Это означает, что

Обрати внимание!

мощность зависит от напряжения.

Рассчитаем мощность лампочки в каждом случае:

 

I=0,2АU=110ВP =U⋅I=110⋅0,2=22(Вт)I=0,4АU=220ВP =U⋅I=220⋅0,4=88(Вт)

 

Можно сделать вывод, что при увеличении напряжения в \(2\) раза мощность увеличивается в \(4\) раза.
Не следует путать эту мощность с номинальной мощностью лампы (мощность, на которую рассчитана лампа). Номинальная мощность лампы (а соответственно — ток через нить накала и её расчётное сопротивление) указываются только для номинального напряжения лампы (указано на баллоне, цоколе или упаковке).


 

 

В таблице дана мощность потребляемая различными приборами и устройствами:

 

Название

Рисунок

Мощность

калькулятор

\(0,001\) Вт

лампы дневного света

\(15 — 80\) Вт

лампы накаливания

\(25 — 5000\) Вт

компьютер

\(200 — 450\) Вт

электрический чайник

\(650 – 3100\) Вт

пылесос

\(1500 — 3000\) Вт

стиральная машина

\(2000 — 4000\) Вт

трамвай

\(150 000 – 240000\) Вт

Источники:

Перышкин А. В Физика. 8 класс // ДРОФА, 2013

http://уроки.мирфизики.рф/%d1%80%d0%b0%d0%b1%d0%be%d1%82%d0%b0-%d0%b8-%d0%bc%d0%be%d1%89%d0%bd%d0%be%d1%81%d1%82%d1%8c-%d1%8d%d0%bb%d0%b5%d0%ba%d1%82%d1%80%d0%b8%d1%87%d0%b5%d1%81%d0%ba%d0%be%d0%b3%d0%be-%d1%82%d0%be%d0%ba/

http://phscs.ru/physicsus/electric-power

http://class-fizika.narod.ru/8_34.htm

Открытый урок физики в 8-м классе по теме «Сила тока»

Тема урока: «Сила тока»

Тип урока: комбинированный (урок объяснения нового материала)

Цели урока:

  • Образовательная: ввести новую физическую величину-силу тока, систематизировать знания через различные виды деятельности;
  • Развивающая: учащиеся устанавливают, что сила тока в различных участках последовательной цепи одинакова, способствовать развитию познавательной деятельности с помощью различных упражнений;
  • Воспитательная: учащиеся убеждаются в необходимости научиться измерять силу тока, способствовать развития мышления, памяти, внимания, культуры письменной и устной речи.

Задачи урока:

  • Образовательная: Выявить имеющиеся знания по данной теме.
  • Воспитательная: Подчеркнуть взаимосвязь силы тока и электрического заряда в электрической цепи.
  • Развивающая: Продолжить работу по развитию внимания и умения логически и творчески мыслить. Продолжить формировать умение решать задачи

Оборудование:

  • мультимедийная приставка,
  • компьютер,
  • две лампы,
  • источник тока,
  • ключ,
  • соединительные провода,
  • раздаточный материал.

Демонстрации: взаимодействие параллельных проводников при замыкании цепи

ХОД УРОКА

I.  Мотивационный-ориентировочный этап

1. Организационный момент

В начале урока обеспечивается рабочая обстановка, организация внимания учеников. Объявляется тема и цель урока.

2. Создание ситуации успеха (проверка домашнего задания)

а) Индивидуальные задания для учащихся

Ученик 1 (работает у доски) Упр. 13(2) Начертить схему цепи, содержащей один гальванический элемент и два звонка, каждый из которых можно включать отдельно
Дополнительное задание: покажите направление тока в данной цепи при замыкании ключа.

Ученик 2 (работает у доски) Упр. 13(5) Нарисуйте схему цепи карманного фонаря (рис. 51 учебника) и назовите части этой цепи. Какие элементы фонаря отмечены цифрами 1-3? Дополнительное задание: Покажите направление упорядоченного движения свободных электронов при замыкании цепи.

Ученики 3, 4. 5

выполняют самостоятельную работу по раздаточному материалу (карточки  1, 2, 3)

Карточка 1

1. Каково направление кратковременного тока, возникающего в проводнике, которым соединены заряженные электроскопы?

______________ О______________
                                                    

2. Как надо поступить, чтобы изменить направление тока в гальванометре?

Карточка 2

1. Назовите, что изображено на схеме? Укажите направление электрического тока на электрической схеме.

1                 2

Карточка 3

Ученик  6 выполняет практическую работу (демонстрационный стол). Собрать электрическую цель по схеме, изображенной на доске.

Дополнительное задание: Как изменить направление тока в цепи?

б) Фронтальный опрос. (Вопросы проектируются на экран с помощью мультимедиа)

Вопросы для проверки:

– Опишите характер движения свободных электронов металле.
– Что представляет собой электрический ток в металлах?
– Проведите сравнение электрического тока с течением воды в водопроводе.
– Какое направление условно приняли за направление электрического тока в проводнике

в) Проверка выполнения индивидуальных заданий.

Учащиеся 3, 4, 5 сдают работы учителю на проверку.

Ученик 1. Схема цепи изображена на доске, показано направление тока. За направление тока принято направление от положительного полюса источника к отрицательному. Поэтому при замыкании цепи направление тока будет таким, как показано стрелками.

Преподаватель: Что произойдет, если один ключ замкнуть, а второй оставить разомкнутым?

Ученик: ‘Га лампа, которая включена последовательно с замкнутым ключом, будет гореть, а вторая нет.

Дополнительные вопросы задают учащиеся.

Ученик  2. Схема цепи карманного фонаря изображена на доске. Упорядоченное движение свободных электронов направлено от отрицательного полюса источника к положительному. Оно указано на чертеже стрелками для случая, когда цепь замкнута. Цифрами 1 обозначены батареи гальванических элементов, 2 –  лампа, 3 –  ключ. Дополнительные вопросы задают учащиеся.

Ученик 6. Цепь состоит из батареи гальванических элементов, двух ламп и ключа. Все приборы соединены последовательно. Чтобы изменить направление электрического тока необходимо поменять провода на клеммах источника тока.

Учащиеся в тетрадях записывают число и изображают схему цепи.

Ученик замыкает цепь.

II. Исполнительский этап

1. Групповая самостоятельная работа

Учащиеся работают с текстом учебника А.В.Перышкин «Физика» 8 кл.

Дискретный подход

Вопросы к ДЭЗ

От чего зависит интенсивность действия электрического тока?
Что такое сила тока?
Как выражается сила тока через электрический заряд и время?
Формула силы тока
Что принимают за единицу силы тока?
Как называется эта единица в СИ?
Какие дольные и кратные амперу единицы силы тока вы знаете?
Получите наименование единицы силы тока
Получите обозначение единицы силы тока
Выведите формулу для вычисления электрического заряда (количества электричества)

 

  ДЭЗ (доминирующий элемент знаний) Источник знаний Вопросы к ДЭЗ
I Интенсивность (степень действия) электрического тока зависит от заряда, прошедшего по цепи в 1с. Стр. 84 От чего зависит интенсивность действия электрического тока?
2 Электрический заряд, проходящий через поперечное сечение проводника в 1с, определяет силу тока в цепи Стр. 85 Что такое сила тока?
3 Сила тока равна отношению электрического заряда q, прошедшего через поперечное сечение проводника, ко времени его прохождения t. Стр. 85 Как выражается сила тока через электрический заряд и время?
4 I = q/t Стр. 85 Формула силы тока
5 За единицы силы тока принимают силу тока, при которой отрезки таких параллельных проводников длиной 1 м взаимодействуют с силой 2*10 Н (0,0000002 Н) Стр. Что принимают за единицу силы тока?
6 В Международной системе (СИ) эту единицу силы тока называют ампером Стр. 86 Как называется эта единица в СИ?
7 Миллиампер (1мА = 0,001А) микроампер (1 мкА = 0,000001А) килоампер (1 кА =  1000А) Стр. 86 Какие дольные и кратные амперу единицы силы тока вы знаете?
8 [1] = 1л]/|т] = заряд/время Стр. 86 Получите наименование единицы силы тока
9 [I] = [q]/[t] = A c =  Кл Стр. 86 Получите обозначение единицы силы тока
10 Q = lt Стр. 87 Выведите формулу для вычисления электрического заряда (количества электричества)

2. Взаимообучение

ДЭЗ (доминирующий элемент знаний)
Интенсивность (степень действия) электрического тока зависит от заряда, прошедшего по цепи в 1с.
Электрический заряд, проходящий через поперечное сечение проводника в 1с, определяет силу тока цепи
Сила тока равна отношению электрического заряда q, прошедшего через поперечное сечение проводника, ко времени его прохождения t.
I = q/t
За единицы силы тока принимают силу тока, при которой отрезки таких параллельных проводников длиной 1 м взаимодействуют с силой 2*10 Н (0,0000002 Н)
В Международной системе (СИ) эту единицу силы тока называют ампером
Миллиампер (1мА = 0,001А) микроампер (1 мкА = 0,000001 А) килоампер (1 кА =  1000А)
[I] = [ q]/[I] = заряд/время
[I] = [q]/ [ t] = A c  =  Кл
Q = I t

 

Обозначение (и назначение) физической величины

Формула для вычисления физической величины

Единицы физической величины

Формула в обобщенном математическом виде

Силатока[I] [I] =  [q]/ [t] А C = А/В
Электрический заряд [q] [q]  = [I] [t] Кл А = ВС
Время [t] [t] = [q]/[I] с В = А/С

Учащиеся выбирают задание, делятся на группы. (Учащиеся работают с текстом учебника А.В.Перышкин «Физика» 8 кл.)

I Группа выясняет, как обозначается сила тока, находит формулу для ее вычисления

II Группа выясняет, что принято за единицу измерения силы тока

III Группа выясняет как измеряют силу тока. От группы выступает 1 ученик.

Первая группа: Т.к. сила тока характеризует электрический заряд, проходящий через поперечное сечение проводника за единицу времени, то для вычисления силы тока необходимо заряд, проходящий через поперечное сечение проводника за промежуток времени t, поделить на этот промежуток времени.
(Обозначение и формула фиксируется на доске и в тетрадях)

Вторая группа: На Международной конференции по мерам и весам в 1948 г. было решено в основу определения единицы силы тока положить явление взаимодействия двух проводников с током. (Рисунок 59 учебника появляется на экране. ) Гибкие проводники при прохождении электрического тока могут притягиваться и отталкиваться. Сила притяжения между проводниками зависит от длины проводников, расстояния между ними, среды в которой они находятся и от силы тока.
За единицу силы тока принимают силу тока, при которой отрезки параллельных проводников длиной 1 м на расстоянии 1 м в вакууме взаимодействуют с силой 2*10″7 Н. Эту единицу называют
ампером, в честь французского физика и математика Андре Мари Ампера. (Его портрет появляется на экране) Обозначение: 1А.

П: Велик ли ток в 1 ампер? Посмотрите на таблицу, вы видите данные технического справочника (изображение на экране).

Сила тока

  • в электрической бритве 0,08 А
  • в карманном радиоприемнике 0,1 А
  • в фонарике 0,3 А
  • в велосипедном генераторе 0,3 А
  • в электрической плитке 3-4 А
  • в двигателе троллейбуса 160-200 А

Системно-функциональный подход (физические величины)

Обозначение (и назначение) физической величины

Формула для вычисления физической величины

Единицы физической величины

Формула в обобщенном математическом виде

Сила тока[1] [I] =  [q]/ [t] А С = А/В
Электрический заряд [q] [q] =  [I] [t] Кл А = ВС
Время [t] [t] = [q]/[I] с В = А/С

 

Обязательные знания о физической величине

Физическая величина сила тока

1. Формула 1 =  g/t, где  I – сила тока, g – электрический заряд, t – время, прохождение заряда через поперечное сечение проводника
2. Словесная I формулировка 1 =  g/t, силой тока называется физическая величина, равная отношению электрического заряда, прошедшая через поперечное сечение проводника ко времени его прохождения
3. Физический смысл 1 =  g/t, сила тока показывает какой электрический заряд, прошедший через поперечное сечение проводника за единицу времени
4. Единица физической величины 1 =  g/t – за единицу силы тока принимают силу тока, при которой отрезки таких параметров проводников длиной 1 м взаимодействие с силой 2–10–3
5. Единица физической величины в СИ 1 =  g/t, за единицу силы тока в СИ принята такая сила тока, при котором тело за одну секунду проходит электрический заряд один Кулон
6. Наименование единицы в СИ [I] =  [g] / [t] =  Кулон/секунду.  =  Кл/с – наименование единицы скорости в СИ кулон в секунду обозначение единиц силы тока – Кл/с

Сила тока, проходящая через тело человека, считающая безопасной 1 мА(0,001 А)
Сила тока, приводящая к серьезным поражениям организма – 100 мА (0,1 А).
Через единицу силы тока определяется единица электрического заряда – 1 Кл.

1 кулон – это электрический заряд, проходящий через поперечное сечение проводника, при силе тока 1 ампер за время 1 секунд

III. Рефлексивно-оценочный этап

– Что же нового вы узнали на уроке?

У: Обозначение силы тока, формулу для ее вычисления, единицы измерения

П: А для чего нужно уметь это делать?

У: В повседневной жизни нас окружают электрические приборы, а мы пользуемся ими, значит необходимо знать, какой должна быть сила тока в электрической цепи, чтобы эти приборы работали в нормальном режиме.

П: На цоколе лампы карманного фонаря написано 0,28 А, что это значит?

У: Это значит, что лампа рассчитана на силу тока не более 0,28 А. ! I:

IV. Объяснение нового домашнего задания: § 37, Упр. 14 (1, 2, 3)

V. Подведение итогов урока

Оценки за урок выставляются учащимся в группах, комментируются. Учитель объявляет оценки за индивидуальную работу по карточкам.

нахождения величины заряда и количество заряда

Напряжения при растяжении сжатии

Используя гипотезу Бернулли для продольной упругой деформации стержня, можно определить продольную силу N как равнодействующую всех рассредоточенных по сечению внутренних усилий. Гипотеза Бернулли совместно с гипотезой о ненадавливании волокон позволяет сказать, что σ в произвольной точке разреза будут постоянны, т.к.  реакция продольных волокон одинакова на всем поперечном разрезе. Для определения величины нормального напряжения σ используется следующая формула:

Напряжение для упруго деформированного стержня описывается как отношение внутренней силы N к площади сечения A. Считается положительным при растяжении, при сжатии рассматривается как отрицательное.

Абсолютная деформация зависит от жесткости сечения, величины продольной силы и длины бруса. Зависимость можно описать по следующей формуле:

Δl=Nl/EA

Таким образом, методика расчета величины абсолютного изменения длины такова: необходимо просчитать отношение значения продольной силы N умноженной на длину стержня l и жесткости сечения (произведение модуля Юнга E на площадь сечения A).

В реальных расчетах на брус действует достаточно много разнонаправленных сил, для решения таких задач требуется построение эпюр, которые могут наглядно показать какие напряжения действуют на разных участках, чем обусловлена деформация при растяжении и сжатии.

В рамках такой квазистатической (условно статической) системы, как брус или стержень с переменным сечением или отверстием, потенциальная энергия растяжения может быть рассмотрена как сумма энергий однородных участков

При проведении расчетов важно правильно разделить стержень на участки и смоделировать все участвующие в процессе силы и напряжения. Для реальных расчетов построение эпюр – сложная задача, которая требует от инженера хорошего понимания действующих на деталь нагрузок

Например, вал со шкивами разного диаметра требует сначала определения критических точек и разбивки на соответствующие участки, затем построения графиков по ним.

Атомная и ядерная физика

Энергия кванта электромагнитной волны (в т.ч. света) или, другими словами, энергия фотона вычисляется по формуле:

Импульс фотона:

Формула Эйнштейна для внешнего фотоэффекта (ЗСЭ):

Максимальная кинетическая энергия вылетающих электронов при фотоэффекте может быть выражена через величину задерживающего напряжение Uз и элементарный заряд е:

Существует граничная частота или длинна волны света (называемая красной границей фотоэффекта) такая, что свет с меньшей частотой или большей длиной волны не может вызвать фотоэффект. Эти значения связаны с величиной работы выхода следующим соотношением:

Второй постулат Бора или правило частот (ЗСЭ):

В атоме водорода выполняются следующие соотношения, связывающие радиус траектории вращающегося вокруг ядра электрона, его скорость и энергию на первой орбите с аналогичными характеристиками на остальных орбитах:

На любой орбите в атоме водорода кинетическая (К) и потенциальная (П) энергии электрона связаны с полной энергией (Е) следующими формулами:

Общее число нуклонов в ядре равно сумме числа протонов и нейтронов:

Дефект массы:

Энергия связи ядра выраженная в единицах СИ:

Энергия связи ядра выраженная в МэВ (где масса берется в атомных единицах):

Формула альфа-распада:

Формула бета-распада:

Закон радиоактивного распада:

Ядерные реакции

Для произвольной ядерной реакции описывающейся формулой вида:

Выполняются следующие условия:

Энергетический выход такой ядерной реакции при этом равен:

Hungarian[edit]

Letteredit


q (lower case, upper case )

  1. The twenty-ninth letter of the Hungarian alphabet, called and written in the Latin script.
Declensionedit

Inflection (stem in long/high vowel, back harmony)
singularplural
nominative
accusative
dative
instrumental
causal-final
translative
terminative
essive-formal
essive-modal
inessive
superessive
adessive
illative
sublative
allative
elative
delative
ablative
non-attributivepossessive — singular
non-attributivepossessive — plural
Possessive forms of q
possessorsingle possessionmultiple possessions
1st person sing. q-m q-im
2nd person sing. q-d q-id
3rd person sing. q-ja q-i
1st person plural q-nk q-ink
2nd person plural q-tok q-itok
3rd person plural q-juk q-ik

(Latin-script letters) betű; A a, Á á, B b, C c, Cs cs, D d, Dz dz, Dzs dzs, E e, É é, F f, G g, Gy gy, H h, I i, Í í, J j, K k, L l, Ly ly, M m, N n, Ny ny, O o, Ó ó, Ö ö, Ő ő, P p, Q q, R r, S s, Sz sz, T t, Ty ty, U u, Ú ú, Ü ü, Ű ű, V v, W w, X x, Y y, Z z, Zs zs

Физический смысл модуля Юнга

Во время принудительного изменения формы предметов внутри них порождаются силы, сопротивляющиеся такому изменению, и стремящиеся к восстановлению исходной формы и размеров упругих тел.

Если же тело не оказывает сопротивления изменению формы и по окончании воздействия остается в деформированном виде, то такое тело называют абсолютно неупругим, или пластичным. Характерным примером пластичного тела является брусок пластилина.

Виды деформации

Р. Гук исследовал удлинение стрежней из различных веществ, под воздействием подвешенных к свободному концу гирь. Количественным выражением степени изменения формы считают относительное удлинение, равное отношению абсолютного удлинения и исходной длины.

В результате серии опытов было установлено, что абсолютное удлинение пропорционально с коэффициентом упругости исходной длине стрежня  и деформирующей силе F и обратно пропорционально площади сечения этого стержня S:

Δl = α * (lF) / S

Величину, обратную α, и называют модулем Юнга:

1/α = E

Относительная деформация:

ε = (Δl) / l = α * (F/S)

Отношение растягивающей силы F к S называют упругим напряжением σ:

ε=α σ

Закон Гука, записанный с использованием модуля Юнга, выглядит так:

σ = ε/α = E ε

Теперь можно сформулировать физический смысл модуля Юнга: он соответствует напряжению, вызываемому растягиванием стержнеобразного образца вдвое, при условии сохранения целостности.

В реальности подавляющее большинство образцов разрушаются до того, как растянутся вдвое от первоначальной длины. Значение E вычисляют с помощью косвенного метода на малых деформациях.

Коэффициент жёсткости при упругой деформации стержня вдоль его оси k = (ES) / l

Модуль Юнга определяет величину потенциальной энергии тел или сред, подвергшихся упругой деформации.

Колебания

Уравнение описывающее физические системы способные совершать гармонические колебания с циклической частотой ω:

Решение предыдущего уравнения является уравнением движения для гармонических колебаний и имеет вид:

Период колебаний вычисляется по формуле:

Частота колебаний:

Циклическая частота колебаний:

Зависимость скорости от времени при гармонических механических колебаниях выражается следующей формулой:

Максимальное значение скорости при гармонических механических колебаниях:

Зависимость ускорения от времени при гармонических механических колебаниях:

Максимальное значение ускорения при механических гармонических колебаниях:

Циклическая частота колебаний математического маятника рассчитывается по формуле:

Период колебаний математического маятника:

Циклическая частота колебаний пружинного маятника:

Период колебаний пружинного маятника:

Максимальное значение кинетической энергии при механических гармонических колебаниях задаётся формулой:

Максимальное значение потенциальной энергии при механических гармонических колебаниях пружинного маятника:

Взаимосвязь энергетических характеристик механического колебательного процесса:

Энергетические характеристики и их взаимосвязь при колебаниях в электрическом контуре:

Период гармонических колебаний в электрическом колебательном контуре определяется по формуле:

Циклическая частота колебаний в электрическом колебательном контуре:

Зависимость заряда на конденсаторе от времени при колебаниях в электрическом контуре описывается законом:

Зависимость электрического тока протекающего через катушку индуктивности от времени при колебаниях в электрическом контуре:

Зависимость напряжения на конденсаторе от времени при колебаниях в электрическом контуре:

Максимальное значение силы тока при гармонических колебаниях в электрическом контуре может быть рассчитано по формуле:

Максимальное значение напряжения на конденсаторе при гармонических колебаниях в электрическом контуре:

Переменный ток характеризуется действующими значениями силы тока и напряжения, которые связаны с амплитудными значениями соответствующих величин следующим образом. Действующее значение силы тока:

Действующее значение напряжения:

Мощность в цепи переменного тока:

Трансформатор

Если напряжение на входе в трансформатор равно U1, а на выходе U2, при этом число витков в первичной обмотке равно n1, а во вторичной n2, то выполняется следующее соотношение:

Коэффициент трансформации вычисляется по формуле:

Если трансформатор идеальный, то выполняется следующее соотношение (мощности на входе и выходе равны):

В неидеальном трансформаторе вводится понятие КПД:

Волны

Длина волны может быть рассчитана по формуле:

Разность фаз колебаний двух точек волны, расстояние между которыми l:

Скорость электромагнитной волны (в т.ч. света) в некоторой среде:

Скорость электромагнитной волны (в т.ч. света) в вакууме постоянна и равна с = 3∙108 м/с, она также может быть вычислена по формуле:

Скорости электромагнитной волны (в т.ч. света) в среде и в вакууме также связаны между собой формулой:

При этом показатель преломления некоторого вещества можно рассчитать используя формулу:

Translingual[edit]

English Wikipedia has an article on:q

Wikipedia

Letteredit


q (upper case )

  1. The seventeenth letter of the basic modern Latin alphabet.
See alsoedit

  • (Latin script):  Aa  Bb  Cc  Dd  Ee  Ff  Gg  Hh  Ii  Jj  Kk  Ll  Mm  Nn  Oo  Pp  Qq  Rr  Sſs  Tt  Uu  Vv  Ww  Xx  Yy  Zz
  • (Variations of letter Q):  Ɋɋ  ʠ  Qq  ȹ
  • With hook: ʠ

.mw-parser-output .k-player .k-attribution{visibility:hidden}IPA

(file)

Symboledit


English Wikipedia has an article on:Voiceless uvular plosive

Wikipedia

q

  1. (IPA) voiceless uvular plosive.
  2. (physics) electrical charge
  3. (physics) heat
  4. (fluid dynamics) dynamic pressure
Synonymsedit

  • (electrical charge)
  • (heat)

See alsoedit


Other representations of Q:

  • NATO phonetic: Quebec
  • Morse code: – – · –
  • Braille: ⠟
  • ASL Manual:
Оцените статью:

Максимальный заряд конденсатора формула.

Принцип работы конденсатора

Вам понадобится

  • — знание емкости или геометрических и физических параметров конденсатора;
  • — знание энергии или заряда на конденсаторе.

Инструкция

Найдите напряжение между пластинами конденсатора, если известна текущая величина накопленной им энергии, а также его емкость. Энергия, запасенная конденсатором, может быть вычислена по формуле W=(C∙U²)/2, где C — емкость, а U — напряжение между пластинами. Таким образом, значение напряжения может быть получено как корень из удвоенного значения энергии, деленного на емкость. То есть, оно будет равно: U=√(2∙W/C).

Энергия, запасенная конденсатором, также может быть вычислена на основании значения содержащегося в нем заряда (количества ) и напряжения между обкладками. Формула, задающая соответствие между этими параметрами, имеет вид: W=q∙U/2 (где q — заряд). Следовательно, зная энергию и , можно вычислить напряжение между его пластинами по формуле: U=2∙W/q. -12 Ф/м), ε — относительная диэлектрическая проницаемость пространства между пластинами (ее можно узнать из физических справочников). Вычислив емкость, рассчитайте напряжение одним из методов, приведенных в шагах 1-3.

Обратите внимание

Для получения корректных результатов при вычислении напряжений между обкладками конденсаторов, перед проведением расчетов приводите значения всех параметров в систему СИ.

Для того чтобы знать, можно ли использовать в том или ином месте схемы конденсатор, следует определить его . Способ нахождения этого параметра зависит от того, каким образом он обозначен на конденсаторе и обозначен ли вообще.

Вам понадобится

  • Измеритель емкости

Инструкция

На крупных конденсаторах емкость обычно обозначена открытым текстом: 0,25 мкФ или 15 uF. В этом случае, способ ее определения тривиален.

На менее крупных конденсаторах (в том , SMD) емкость двумя или тремя цифрами. В первом случае, она обозначена в пикофарадах. Во втором случае, первые две цифры емкость , а третья — в каких единицах она выражена:1 — десятки пикофарад;
2 — сотни пикофарад;
3 — нанофарады;
4 — десятки нанофарад;
5 — доли микрофарады.

Существует также система обозначения емкости, использующая сочетания латинских букв и цифр. Буквы обозначают следующие цифры:A — 10;
B — 11;
C — 12;
D — 13;
E — 15;
F — 16;
G — 18;
H — 20;
J — 22;
K — 24;
L — 27;
M — 30;
N — 33;
P — 36;
Q — 39;
R — 43;
S — 47;
T — 51;
U — 56;
V — 62;
W — 68;
X — 75;
Y — 82;
Z — 91.Полученное число следует умножить на число 10, предварительно возведенное в степень, равную цифре, следующей после . Результат будет выражен в пикофарадах.

Встречаются конденсаторы, емкость на которых не обозначена вообще. Вы наверняка встречали их, в , в стартерах ламп дневного . В этом случае, измерить емкость можно только специальным прибором. Они цифровыми и мостовыми.В любом случае, если конденсатор впаян в то или иное устройство, его следует обесточить, разрядить в нем конденсаторы фильтра и сам конденсатор, емкость которого следует измерить, и лишь после этого выпаять его. Затем его необходимо подключить к прибору.На цифровом измерителе сначала выбирают самый грубый предел, затем переключают его до тех пор, пока он не покажет перегрузку. После этого переключатель переводят на один предел назад и читают показания, а по положению переключателя определяют, в каких единицах они выражены.На мостовом измерителе, последовательно переключая , на каждом из них прокручивают регулятор из одного конца шкалы в другой, пока звук из динамика не исчезнет. Добившись исчезновения , по шкале регулятора считывают результат, а единицы, в которых он выражен, также определяют по положению переключателя.Затем конденсатор устанавливают обратно в устройство.

Обратите внимание

Никогда не подключайте к измерителю заряженные конденсаторы.

Источники:

  • Справочник по системам обозначения емкости

Найти значение электрического заряда можно двумя способами. Первый – измерить силу взаимодействия неизвестного заряда с известным и с помощью закона Кулона рассчитать его значение. Второй – внести заряд в известное электрическое поле и измерить силу, с которой оно действует на него. Для измерения заряда протекающего через поперечное сечение проводника за определенное время измерьте силу тока и умножьте ее на значение времени.

Вам понадобится

  • чувствительный динамометр, секундомер, амперметр, измеритель электростатического поля, воздушный конденсатор.

Инструкция

Измерение заряда при его с известным зарядомЕсли известен одного тела, поднесите к нему неизвестный заряд и измерьте между ними в метрах. Заряды начнут взаимодействовать. С помощью динамометра измерьте силу их взаимодействия. Рассчитайте значение неизвестного заряда — для этого квадрат измеренного расстояния умножьте на значение силы и поделите на известный заряд. 9)). Если заряды отталкиваются, то они одноименные, если же притягиваются – разноименные.

Измерение значения заряда , внесенного в электрическое полеИзмерьте значение постоянного электрического поля специальным прибором (измеритель электрического поля). Если такого прибора нет, возьмите воздушный конденсатор, зарядите его, измерьте напряжение на его обкладках и поделите не расстояние между пластинами – это и будет значение электрического поля внутри конденсатора в вольтах на метр. Внесите в поле неизвестный заряд. С помощью чувствительного динамометра измерьте силу, которая на него действует. Измерение проводите в . Поделите значение силы на напряженность электрического поля. Результатом будет значение заряда в Кулонах (q=F/Е).

Измерение заряда , протекающего через поперечное проводникаСоберите электрическую цепь с проводниками и последовательно подключите к ней амперметр. Замкните ее на источник тока и измерьте силу тока с помощью амперметра в амперах. Одновременно секундомером засеките , в которого в цепи был электрический ток. Умножив значение силы тока на полученное время, узнайте заряд, через поперечное сечение каждого за это время (q=I t). При измерениях следите, чтобы проводники не перегревались и не произошло короткое замыкание.

Конденсатором называется устройство, способное накапливать электрические заряды. Количество накапливаемой электрической энергии в конденсаторе характеризуется его емкостью . Она измеряется в фарадах. Считается, что емкость в один фарад соответствует конденсатору, заряженному электрическим зарядом в один кулон при разности потенциалов на его обкладках в один вольт.

Инструкция

Определите емкость плоского конденсатора по формуле С = S e e0/d, где S — площадь поверхности одной пластины, d — между пластинами, e — относительная диэлектрическая проницаемость , заполняющей пространство между пластинами (в вакууме она равна ), e0 — электрическая постоянная, равная 8,854187817 10(-12) Ф/м. Исходя из приведенной формулы, величина емкости будет зависеть от площади проводников, между ними и от материала диэлектрика. В качестве диэлектрика может применяться или слюда.

Вычислите емкость сферического конденсатора по формуле С = (4П e0 R²)/d, где П — число «пи», R — радиус сферы, d — величина зазора между его сферами.Величина емкости сферического конденсатора прямо пропорциональна концентрической сферы и обратно пропорциональна расстоянию между сферами.

Рассчитайте емкость цилиндрического конденсатора по формуле С = (2П e e0 L R1)/(R2-R1), где L — длина конденсатора , П — число «пи», R1 и R2 — радиусы его цилиндрических обкладок.

Если конденсаторы в цепи соединены параллельно, рассчитайте их общую емкость по формуле С = С1+С2+…+Сn, где С1, С2,…Сn – емкости параллельно соединенных конденсаторов.

Вычислите общую емкость последовательно соединенных конденсаторов по формуле 1/С = 1/С1+1/С2+…+1/Сn, где С1, С2,…Сn — емкости последовательно соединенных конденсаторов.

Обратите внимание

На любом конденсаторе обязательно должна быть нанесена маркировка, которая может быть буквенно-цифровая или цветовая. Маркировка отражает его параметры.

Источники:

  • Цветовая маркировка резисторов, конденсаторов и индуктивностей

Емкость – величина, в системе СИ выражаемая в фарадах. Хотя используются, фактически, лишь производные от нее – микрофарады, пикофарады и так далее. Что касается электроемкости плоского конденсатора, она зависит от зазора меж обкладок и их площади, от вида диэлектрика, в данном зазоре расположенного.

Инструкция

В том случае, если обкладки конденсатора имеют одинаковую площадь и имеют расположение строго одна над другой, рассчитайте площадь одной из обкладок – любой. Если же одна из них относительно другой смещена либо они разные , нужно рассчитывать площадь области, в которой обкладки друг дружку перекрывают.

При этом используются общепринятые формулы, рассчитывать площади таких геометрических фигур, как круг (S=π(R^2)), прямоугольника (S=ab), его частного случая – квадрата (S=a^2) – и других. (-12) Ф/м и является, по сути, диэлектрической проницаемостью вакуума.

Темы кодификатора ЕГЭ : электрическая ёмкость, конденсатор, энергия электрического поля конденсатора.

Предыдущие две статьи были посвящены отдельному рассмотрению того, каким образом ведут себя в электрическом поле проводники и каким образом — диэлектрики. Сейчас нам понадобится объединить эти знания. Дело в том, что большое практическое значение имеет совместное использование проводников и диэлектриков в специальных устройствах — конденсаторах .

Но прежде введём понятие электрической ёмкости .

Ёмкость уединённого проводника

Предположим, что заряженный проводник расположен настолько далеко от всех остальных тел, что взаимодействие зарядов проводника с окружающими телами можно не принимать во внимание. В таком случае проводник называется уединённым .

Потенциал всех точек нашего проводника, как мы знаем, имеет одно и то же значение , которое называется потенциалом проводника. Оказывается, что потенциал уединённого проводника прямо пропорционален его заряду . Коэффициент пропорциональности принято обозначать , так что

Величина называется электрической ёмкостью проводника и равна отношению заряда проводника к его потенциалу:

(1)

Например, потенциал уединённого шара в вакууме равен:

где — заряд шара, — его радиус. Отсюда ёмкость шара:

(2)

Если шар окружён средой-диэлектриком с диэлектрической проницаемостью , то его потенциал уменьшается в раз:

Соответственно, ёмкость шара в раз увеличивается:

(3)

Увеличение ёмкости при наличии диэлектрика — важнейший факт. Мы ещё встретимся с ним при рассмотрении конденсаторов.

Из формул (2) и (3) мы видим, что ёмкость шара зависит только от его радиуса и диэлектрической проницаемости окружающей среды. То же самое будет и в общем случае: ёмкость уединённого проводника не зависит от его заряда; она определяется лишь размерами и формой проводника, а также диэлектрической проницаемостью среды, окружающей проводник. От вещества проводника ёмкость также не зависит.

В чём смысл понятия ёмкости? Ёмкость показывает, какой заряд нужно сообщить проводнику, чтобы увеличить его потенциал на В . Чем больше ёмкость — тем, соответственно, больший заряд требуется поместить для этого на проводник.

Единицей измерения ёмкости служит фарад (Ф). Из определения ёмкости (1) видно, что Ф = Кл/В.

Давайте ради интереса вычислим ёмкость земного шара (он является проводником!). Радиус считаем приближённо равным км.

МкФ.

Как видите, Ф — это очень большая ёмкость.

Единица измерения ёмкости полезна ещё и тем, что позволяет сильно сэкономить на обозначении размерности диэлектрической постоянной . В самом деле, выразим из формулы (2) :

Следовательно, диэлектрическая постоянная может измеряться в Ф/м:

Так легче запомнить, не правда ли?

Ёмкость плоского конденсатора

Ёмкость уединённого проводника на практике используется редко. В обычных ситуациях проводники не являются уединёнными. Заряженный проводник взаимодействует с окружающими телами и наводит на них заряды, а потенциал поля этих индуцированных зарядов (по принципу суперпозиции!) изменяет потенциал самого проводника. В таком случае уже нельзя утверждать, что потенциал проводника будет прямо пропорционален его заряду, и понятие ёмкости проводника самого по себе фактически утрачивает смысл.

Можно, однако, создать систему заряженных проводников, которая даже при накоплении на них значительного заряда почти не взаимодействует с окружающими телами. Тогда мы сможем снова говорить о ёмкости — но на сей раз о ёмкости этой системы проводников.

Наиболее простым и важным примером такой системы является плоский конденсатор . Он состоит из двух параллельных металлических пластин (называемых обкладками ), разделённых слоем диэлектрика. При этом расстояние между пластинами много меньше их собственных размеров.

Для начала рассмотрим воздушный конденсатор, у которого между обкладками находится воздух

Пусть заряды обкладок равны и . Именно так и бывает в реальных электрических схемах: заряды обкладок равны по модулю и противоположны по знаку. Величина — заряд положительной обкладки — называется зарядом конденсатора .

Пусть — площадь каждой обкладки. Найдём поле, создаваемое обкладками в окружающем пространстве.

Поскольку размеры обкладок велики по сравнению с расстоянием между ними, поле каждой обкладки вдали от её краёв можно считать однородным полем бесконечной заряженной плоскости:

Здесь — напряжённость поля положительной обкладки, — напряженность поля отрицательной обкладки, — поверхностная плотность зарядов на обкладке:

На рис. 1 (слева) изображены векторы напряжённости поля каждой обкладки в трёх областях: слева от конденсатора, внутри конденсатора и справа от конденсатора.

Рис. 1. Электрическое поле плоского конденсатора

Согласно принципу суперпозиции, для результирующего поля имеем:

Нетрудно видеть, что слева и справа от конденсатора поле обращается в нуль (поля обкладок погашают друг друга):

Внутри конденсатора поле удваивается:

(4)

Результирующее поле обкладок плоского конденсатора изображено на рис. 1 справа. Итак:

Внутри плоского конденсатора создаётся однородное электрическое поле, напряжённость которого находится по формуле (4) . Снаружи конденсатора поле равно нулю, так что конденсатор не взаимодействует с окружающими телами.

Не будем забывать, однако, что данное утверждение выведено из предположения, будто обкладки являются бесконечными плоскостями. На самом деле их размеры конечны, и вблизи краёв обкладок возникают так называемые краевые эффекты : поле отличается от однородного и проникает в наружное пространство конденсатора. Но в большинстве ситуаций (и уж тем более в задачах ЕГЭ по физике) краевыми эффектами можно пренебречь и действовать так, словно утверждение, выделенное курсивом, является верным без всяких оговорок.

Пусть расстояние между обкладками конденсатора равно . Поскольку поле внутри конденсатора является однородным, разность потенциалов между обкладками равна произведению на (вспомните связь напряжения и напряжённости в однородном поле!):

(5)

Разность потенциалов между обкладками конденсатора, как видим, прямо пропорциональна заряду конденсатора. Данное утверждение аналогично утверждению «потенциал уединённого проводника прямо пропорционален заряду проводника», с которого и начался весь разговор о ёмкости. Продолжая эту аналогию, определяем ёмкость конденсатора как отношение заряда конденсатора к разности потенциалов между его обкладками:

(6)

Ёмкость конденсатора показывает, какой заряд ему нужно сообщить, чтобы разность потенциалов между его обкладками увеличилась на В. Формула (6) , таким образом, является модификацией формулы (1) для случая системы двух проводников — конденсатора.

Из формул (6) и (5) легко находим ёмкость плоского воздушного конденсатора :

(7)

Она зависит только от геометрических характеристик конденсатора: площади обкладок и расстояния между ними.
Предположим теперь, что пространство между обкладками заполнено диэлектриком с диэлектрической проницаемостью . Как изменится ёмкость конденсатора?

Напряжённость поля внутри конденсатора уменьшится в раз, так что вместо формулы (4) теперь имеем:

(8)

Соответственно, напряжение на конденсаторе:

(9)

Отсюда ёмкость плоского конденсатора с диэлектриком :

(10)

Она зависит от геометрических характеристик конденсатора (площади обкладок и расстояния между ними) и от диэлектрической проницаемости диэлектрика, заполняющего конденсатор.

Важное следствие формулы (10) : заполнение конденсатора диэлектриком увеличивает его ёмкость .

Энергия заряженного конденсатора

Заряженный конденсатор обладает энергией. В этом можно убедиться на опыте. Если зарядить конденсатор и замкнуть его на лампочку, то (при условии, что ёмкость конденсатора достаточно велика) лампочка ненадолго загорится.

Следовательно, в заряженном конденсаторе запасена энергия, которая и выделяется при его разрядке. Нетрудно понять, что этой энергией является потенциальная энергия взаимодействия обкладок конденсатора — ведь обкладки, будучи заряжены разноимённо, притягиваются друг к другу.

Мы сейчас вычислим эту энергию, а затем увидим, что существует и более глубокое понимание происхождения энергии заряженного конденсатора.

Начнём с плоского воздушного конденсатора. Ответим на такой вопрос: какова сила притяжения его обкладок друг к другу? Величины используем те же: заряд конденсатора , площадь обкладок .

Возьмём на второй обкладке настолько маленькую площадку, что заряд этой площадки можно считать точечным. Данный заряд притягивается к первой обкладке с силой

где — напряжённость поля первой обкладки:

Следовательно,

Направлена эта сила параллельно линиям поля (т. е. перпендикулярно пластинам).

Результирующая сила притяжения второй обкладки к первой складывается из всех этих сил , с которыми притягиваются к первой обкладке всевозможные маленькие заряды второй обкладки. При этом суммировании постоянный множитель вынесется за скобку, а в скобке просуммируются все и дадут . В результате получим:

(11)

Предположим теперь, что расстояние между обкладками изменилось от начальной величины до конечной величины . Сила притяжения пластин совершает при этом работу:

Знак правильный: если пластины сближаются , то сила совершает положительную работу, так как пластины притягиваются друг к другу. Наоборот, если удалять пластины alt=»(d_2 > d_1)»> , то работа силы притяжения получается отрицательной, как и должно быть.

С учётом формул (11) и (7) имеем:

Это можно переписать следующим образом:

(12)

Работа потенциальной силы притяжения обкладок оказалась равна изменению со знаком минус величины . Это как раз и означает, что — потенциальная энергия взаимодействия обкладок, или энергия заряженного конденсатора .

Используя соотношение , из формулы (12) можно получить ещё две формулы для энергии конденсатора (убедитесь в этом самостоятельно!):

(13)

(14)

Особенно полезными являются формулы (12) и (14) .

Допустим теперь, что конденсатор заполнен диэлектриком с диэлектрической проницаемостью . Сила притяжения обкладок уменьшится в раз, и вместо (11) получим:

При вычислении работы силы , как нетрудно видеть, величина войдёт в ёмкость , и формулы (12) — (14) останутся неизменными . Ёмкость конденсатора в них теперь будет выражаться по формуле (10) .

Итак, формулы (12) — (14) универсальны: они справедливы как для воздушного конденсатора, так и для конденсатора с диэлектриком.

Энергия электрического поля

Мы обещали, что после вычисления энергии конденсатора дадим более глубокое истолкование происхождения этой энергии. Что ж, приступим.

Рассмотрим воздушный конденсатор и преобразуем формулу (14) для его энергии:

Но — объём конденсатора. Получаем:

(15)

Посмотрите внимательно на эту формулу. Она уже не содержит ничего, что являлось бы специфическим для конденсатора! Мы видим энергию электрического поля , сосредоточенного в некотором объёме .

Энергия конденсатора есть не что иное, как энергия заключённого внутри него электрического поля.

Итак, электрическое поле само по себе обладает энергией. Ничего удивительного для нас тут нет. Радиоволны, солнечный свет — это примеры распространения энергии, переносимой в пространстве электромагнитными волнами.

Величина — энергия единицы объёма поля — называется объёмной плотностью энергии . Из формулы (15) получим:

(16)

В этой формуле не осталось вообще никаких геометрических величин. Она даёт максимально чистую связь энергии электрического поля и его напряжённости.

Если конденсатор заполнен диэлектриком, то его ёмкость увеличивается в раз, и вместо формул (15) и (16) будем иметь:

(17)

(18)

Как видим, энергия электрического поля зависит ещё и от диэлектрической проницаемости среды, в которой поле находится.
Замечательно, что полученные формулы для энергии и плотности энергии выходят далеко за пределы электростатики: они справедливы не только для электростатического поля, но и для электрических полей, меняющихся во времени.

Одними из наиболее часто используемых электронных компонентов являются конденсаторы . И в этой статье нам предстоит разобраться, из чего они состоят, как работают и для чего применяются 🙂

Давайте, в первую очередь, рассмотрим устройство конденсаторов , а затем уже плавно перейдем к их основным видам и характеристикам, а также к процессам зарядки/разрядки. Как видите, нам сегодня предстоит изучить много интересных моментов 😉

Итак, простейший конденсатор представляет из себя две плоские проводящие пластины, расположенные параллельно друг другу и разделенные слоем диэлектрика. Причем расстояние между пластинами должно быть намного меньше, чем, собственно, размеры пластин:

Такое устройство называется плоским конденсатором , а пластины – обкладками конденсатора . Стоит уточнить, что здесь мы рассматриваем уже заряженный конденсатор (сам процесс зарядки мы изучим чуть позже), то есть на обкладках сосредоточен определенный заряд. Причем наибольший интерес представляет тот случай, когда заряды пластин конденсатора одинаковы по модулю и противоположны по знаку (как на рисунке).

А поскольку на обкладках сосредоточен заряд, между ними возникает электрическое поле, изображенное стрелками на нашей схеме. Поле плоского конденсатора, в основном, сосредоточено между пластинами, однако, в окружающем пространстве также возникает электрическое поле, которое называют полем рассеяния. Очень часто его влиянием в задачах пренебрегают, но забывать о нем не стоит 🙂

Для определения величины этого поля рассмотрим еще одно схематическое изображение плоского конденсатора:

Каждая из обкладок конденсатора в отдельности создает электрическое поле:

Выражение для напряженности поля равномерно заряженной пластины выглядит следующим образом:

Здесь – это поверхностная плотность заряда: . А – диэлектрическая проницаемость диэлектрика, расположенного между обкладками конденсатора. Поскольку площадь пластин конденсатора у нас одинаковая, как и величина заряда, то и модули напряженности электрического поля, равны между собой:

Но направления векторов разные – внутри конденсатора вектора направлены в одну сторону, а вне – в противоположные. Таким образом, внутри обкладок результирующее поле определяется следующим образом:

А какая же будет величина напряженности вне конденсатора? А все просто – слева и справа от обкладок поля пластин компенсируют друг друга и результирующая напряженность равна 0 🙂

Процессы зарядки и разрядки конденсаторов.

С устройством мы разобрались, теперь разберемся, что произойдет, если подключить к конденсатору источник постоянного тока. На принципиальных электрических схемах конденсатор обозначают следующим образом:

Итак, мы подключили обкладки конденсатора к полюсам источника постоянного тока. Что же будет происходить?

Свободные электроны с первой обкладки конденсатора устремятся к положительному полюсу источника, в связи с чем на обкладке возникнет недостаток отрицательно заряженных частиц и она станет положительно заряженной. В то же время электроны с отрицательного полюса источника тока переместятся ко второй обкладке конденсатора, в результате чего на ней возникнет избыток электронов, соответственно, обкладка станет отрицательно заряженной. Таким образом, на обкладках конденсатора образуются заряды разного знака (как раз этот случай мы и рассматривали в первой части статьи), что приводит к появлению электрического поля, которое создаст между пластинами конденсатора определенную . Процесс зарядки будет продолжаться до тех пор, пока эта разность потенциалов не станет равна напряжению источника тока, после этого процесс зарядки закончится, и перемещение электронов по цепи прекратится.

При отключении от источника конденсатор может на протяжении длительного времени сохранять накопленные заряды. Соответственно, заряженный конденсатор является источником электрической энергии, это означает, что он может отдавать энергию во внешнюю цепь. Давайте создадим простейшую цепь, просто соединив обкладки конденсатора друг с другом:

В данном случае по цепи начнет протекать ток разряда конденсатора , а электроны начнут перемещаться с отрицательно заряженной обкладки к положительной. В результате напряжение на конденсаторе (разность потенциалов между обкладками) начнет уменьшаться. Этот процесс завершится в тот момент, когда заряды пластин конденсаторов станут равны друг другу, соответственно электрическое поле между обкладками пропадет и по цепи перестанет протекать ток. Вот так и происходит разряд конденсатора, в результате которого он отдает во внешнюю цепь всю накопленную энергию.

Как видите, здесь нет ничего сложного 🙂

Емкость и энергия конденсатора.

Важнейшей характеристикой является электрическая емкость конденсатора – физическая величина, которая определяется как отношение заряда конденсатора одного из проводников к разности потенциалов между проводниками:

Емкость изменяется в Фарадах, но величина 1 Ф является довольно большой, поэтому чаще всего емкость конденсаторов измерятся в микрофарадах (мкФ), нанофарадах (нФ) и пикофарадах (пФ).

А поскольку мы уже вывели формулу для расчета напряженности, то давайте выразим напряжение на конденсаторе следующим образом:

Здесь у нас – это расстояние между пластинами конденсатора, а – заряд конденсатора. Подставим эту формулу в выражение для емкости конденсатора:

Если в качестве диэлектрика у нас выступает воздух, то во всех формулах можно подставить

Для запасенной энергии конденсатора справедливы следующие выражения:

Помимо емкости конденсаторы характеризуются еще одним параметром, а именно величиной напряжения, которое может выдержать его диэлектрик. При слишком больших значениях напряжения электроны диэлектрика отрываются от атомов, и диэлектрик начинает проводить ток. Это явление называется пробоем конденсатора, и в результате обкладки оказываются замкнутыми друг с другом. Собственно, характеристикой, которая часто используется при работе с конденсаторами является не напряжение пробоя, а рабочее напряжение – то есть величина напряжения, при которой конденсатор может работать неограниченно долгое время, и пробоя не произойдет.

В общем, мы рассмотрели сегодня основные свойства конденсаторов, их устройство и характеристики, так что на этом заканчиваем статью, а в следующей мы будем обсуждать различные варианты соединений конденсаторов, так что заходите на наш сайт снова!

Как и любая система заряжен-ных тел, конденсатор обладает энер-гией. Вычислить энергию заряжен-ного плоского конденсатора с одно-родным полем внутри него не-сложно.

Энергия заряженного конденса-тора.

Для того чтобы зарядить конденсатор, нужно совершить рабо-ту по разделению положительных и отрицательных зарядов. Согласно закону сохранения энергии эта ра-бота равна энергии конденсатора. В том, что заряженный конденсатор обладает энергией, можно убедиться, если разрядить его через цепь, со-держащую лампу накаливания, рас-считанную на напряжение в не-сколько вольт (рис. 4). При раз-рядке конденсатора лампа вспыхи-вает. Энергия конденсатора пре-вращается в другие формы: тепло-вую, световую.

Выведем формулу для энергии плоского конденсатора .

Напряженность поля, созданного зарядом одной из пластин, равна Е/2, где Е — напряженность поля в конденсаторе. В однородном поле одной пластины находится заряд q, распределенный по поверхности дру-гой пластины (рис. 5). Согласно формуле W p = qEd. для потенциальной энергии заряда в однородном поле энергия конденсатора равна:

Можно доказать, что эти форму-лы справедливы для энергии любого конденсатора, а не только для плос-кого.

Энергия электрического поля.

Согласно теории близкодействия вся энергия взаимодействия заряженных тел сконцентрирована в электриче-ском поле этих тел. Значит, энергия может быть выражена через основную характеристику поля — напря-женность.

Так как напряженность электри-ческого поля прямо пропорциональ-на разности потенциалов

(U = Ed), то согласно формуле

энергия конденсатора прямо пропор-циональна напряженности электри-ческого поля внутри него: W p ~ E 2 . Детальный расчет дает следующее значение для энергии поля, приходя-щейся на единицу объема, т.е. для плотности энергии:

где ε 0 — электрическая постоянная

Применение конденсаторов.

Энер-гия конденсатора обычно не очень велика — не более сотен джоулей. К тому же она не сохраняется долго из-за неизбежной утечки заряда. Поэтому заряженные конденсаторы не могут заменить, например, акку-муляторы в качестве источников электрической энергии.

Но это совсем не означает, что конденсаторы как накопители энергии не получили практического при-менения. Они имеют одно важное свойство: конденсаторы могут на-капливать энергию более или менее длительное время, а при разрядке через цепь малого сопротивления они отдают энергию почти мгновенно. Именно это свойство используют широко на практике.

Лампа-вспышка, применяемая в фотографии, питается электрическим током разряда конденсатора, заря-жаемого предварительно специаль-ной батареей. Возбуждение кванто-вых источников света — лазеров осу-ществляется с помощью газораз-рядной трубки, вспышка которой происходит при разрядке батареи конденсаторов большой электроем-кости.

Однако основное применение кон-денсаторы находят в радиотехнике. С этим вы познакомитесь в XI классе.

Энергия конденсатора пропор-циональна его электроемкости и квадрату напряжения между плас-тинами. Вся эта энергия сосредото-чена в электрическом поле. Плот-ность энергии поля пропорциональна квадрату напряженности поля.

Рис. 1 Рис. 2

ЗАКОНЫ ПОСТОЯННОГО ТОКА.

Неподвижные электрические заряды редко используются на практике. Для того чтобы заставить электрические заряды слу-жить нам, их нужно привести в движение — создать электрический ток. Электрический ток освещает квартиры, приводит в дви-жение станки, создает радиоволны, циркулирует во всех электрон-но-вычислительных машинах.

Мы начнем с наиболее простого случая движения заряжен-ных частиц — рассмотрим постоянный электрический ток.

ЭЛЕКТРИЧЕСКИЙ ТОК. СИЛА ТОКА

Дадим строгое определение тому, что называют электрическим током.

Напомним, какой величиной ха-рактеризуется ток количественно.

Найдем, как быстро движутся электроны по проводам в вашей квартире.

При движении заряженных час-тиц в проводнике происходит перенос электрического заряда с одного места в другое. Однако если заряженные частицы совершают беспорядочное тепловое движение, как, например, свободные электроны в металле, то переноса заряда не про-исходит (рис.1). Электриче-ский заряд перемещается через по-перечное сечение проводника лишь в том случае, если наряду с беспорядочным движением электроны участвуют в упорядоченном движении (рис. 2). В этом случае говорят, что в проводнике устанавливается электрический ток.

Из курса физики VIII класса вы знаете, что электрическим током называют упорядоченное (направ-ленное) движение заряженных частиц.

Электрический ток возникает при упорядоченном перемещении свобод-ных электронов или ионов.

Если перемещать нейтральное в целом тело, то, несмотря на упо-рядоченное движение огромного чис-ла электронов, и атомных ядер, электрический ток не возникает. Полный заряд, переносимый через любое сечение проводника, будет при этом равным нулю, так как заряды разных знаков с одинаковой средней скоростью.

Электрический ток имеет определенное направление. За направление тока принимают направление движения положительно заряженных частиц. Если ток образован движением отрицательно заряженных частиц, то направление тока считают противоположным направлению дви-жения частиц.

Действия тока. Движение частиц в проводнике мы непосредственно не видим. О наличии электрического тока приходится судить по тем дей-ствиям или явлениям, которые его сопровождают.

Во-первых, проводник, по которо-му течет ток, нагревается.

Во-вторых, электрический ток мо-жет изменять химический состав проводника, например, выделять его химические составные части (медь из раствора медного купороса и т.д.).

В-третьих, ток оказывает силовое воздействие на соседние токи и на-магниченные тела. Это действие то-ка называется магнитным. Так, маг-нитная стрелка вблизи проводника с током поворачивается. Магнитное действие тока в отличие от химиче-ского и теплового является основ-ным, так как проявляется у всех без исключения проводников. Хими-ческое действие тока наблюдается лишь у растворов и расплавов электролитов, а нагревание отсут-ствует у сверхпроводников.

Сила тока.

Если в цепи уста-навливается электрический ток, то это означает, что через поперечное сечение проводника все время пере-носится электрический заряд. Заряд, перенесенный в единицу времени, служит основной количественной ха-рактеристикой тока, называемой си-лой тока.

Таким образом, сила тока равна отношению заряда q, переносимого через поперечное сечение провод-ника за интервал времени t, к этому интервалу времени. Если сила тока со временем не меняется, то ток на-зывают постоянным.

Сила тока, подобно заряду, ве-личина скалярная. Она может быть как положительной, так и отрица-тельной. Знак силы тока зависит от того, какое из направлений вдоль проводника принять за положитель-ное. Сила тока / > 0, если направ-ление тока совпадает с условно вы-бранным положительным направле-нием вдоль проводника. В против-ном случае /

Сила тока зависит от заряда, переносимого каждой частицей, кон-центрации частиц, скорости их направленного движения и площади поперечного сечения проводника. По-кажем это.

Пусть проводник (рис. 3) имеет поперечное сечение площадью S. За положительное направление в проводнике примем направление сле-ва направо. Заряд каждой частицы равен q 0 . В объеме проводника, ограниченном поперечными сечениям-и 1 и 2, содержится nSl частиц, где п — концентрация частиц. Их общий заряд q = q Q nSl. Если частицы движутся слева направо со средней скоростью υ, то за время

Все частицы, заключенные в рассматриваемом объеме, пройдут через поперечное сечение 2. Поэтому сила тока равна:

формуле (2) где е — модуль заряда электрона.

Пусть, например, сила тока I = 1 А, а площадь по-перечного сечения проводника S = 10 -6 м 2 . Модуль заряда электрона е = 1,6 — 10 -19 Кл. Число электронов в 1 м 3 меди равно числу атомов в этом объеме, так как один из ва-лентных электронов каждого атома меди коллективизирован и является свободным. Это число есть п = 8,5 · 10 28 м -3 Следовательно,

Рис №1. Рис №2 Рис №3

УСЛОВИЯ, НЕОБХОДИМЫЕ ДЛЯ СУЩЕСТВОВАНИЯ ЭЛЕКТРИЧЕСКОГО ТОКА

Что необходимо для создания электрического тока? Подумайте над этим сами и только потом прочтите этот параграф.

Для возникновения и существо-вания постоянного электрического тока в веществе необходимо, во-первых, наличие свободных заряжен-ных частиц. Если положительные и отрицательные заряды связаны друг с другом в атомах или молекулах , то их перемещение не приведет к по-явлению электрического тока.

Наличия свободных зарядов еще недостаточно для возникновения то-ка. Для создания и поддержания упорядоченного движения, заряжен-ных частиц необходима, во-вторых, сила, действующая на них в опре-деленном направлении. Если эта сила перестанет действовать, то упорядоченное движение заряженных частиц прекратится из-за сопротив-ления, оказываемого их движению ионами кристаллической решетки металлов или нейтральными молеку-лами электролитов .

На заряженные частицы, как мы знаем, действует электрическое поле с силой . Обычно именно электрическое поле внутри провод-ника служит причиной, вызываю-щей и поддерживающей упорядочен-ное движение заряженных частиц. Только в статическом случае, когда заряды покоятся, электрическое поле внутри проводника равно нулю.

Если внутри проводника имеется электрическое поле, то между конца-ми проводника в соответствии с фор-мулой существует разность потенциалов. Когда разность потен-циалов не меняется во времени, то в проводнике устанавливается по-стоянный электрический ток. Вдоль проводника потенциал уменьшается от максимального значения на одном конце проводника до минималь-ного — на другом. Это уменьшение потенциала можно обнаружить на простом опыте.

Возьмем в качестве проводника не очень сухую деревянную палку и подвесим ее горизонтально. (Такая палка хотя и плохо, но все же про-водит ток.) Источником напряжения пусть будет электростатическая ма-шина, Для регистрации потенциала различных участков проводника от-носительно земли можно использо-вать листочки металлической фоль-ги, прикрепленные к палке. Один полюс машины соединим с землей, а второй — с одним концом проводни-ка (палки). Цепь окажется незамк-нутой. При вращении рукоятки ма-шины мы обнаружим, что все лис-точки отклоняются на один и тот же угол (рис. 1).

Значит, потен-циал всех точек проводника отно-сительно земли одинаков. Так и должно быть при равновесии заря-дов на проводнике. Если теперь дру-гой конец палки заземлить, то при вращении рукоятки машины карти-на изменится. (Так как земля — проводник, то заземление провод-ника делает цепь замкнутой.) У за-земленного конца листочки вообще не разойдутся: потенциал этого кон-ца проводника практически равен потенциалу земли (падение потен-циала в металлической проволоке мало). Максимальный угол расхож-дения листочков будет у конца про-водника, присоединенного к машине (рис. 2). Уменьшение угла рас-хождения листочков по мере удале-ния от машины свидетельствует о падении потенциала вдоль провод-ника.

Электрический ток может быть получен только в веществе, в котором имеются свободные заряженные частицы. Чтобы они пришли в движение, нужно создать в проводнике электрическое поле.

Рис №1 Рис №2

ЗАКОН ОМА ДЛЯ УЧАСТКА ЦЕПИ. СОПРОТИВЛЕНИЕ

В VIII классе изучался закон Ома . Этот закон прост, однако столь важен, что его необходимо повторить.

Вольт — амперная характеристика.

В предыдущем параграфе было уста-новлено, что для существования то-ка в проводнике необходимо создать разность потенциалов на его концах. Сила тока в проводнике определяет-ся этой разностью потенциалов. Чем больше разность потенциалов, тем больше напряженность электриче-ского поля в проводнике и, следо-вательно, тем большую скорость на-правленного движения приобретают заряженные частицы. Согласно фор-муле, это означает увеличение силы тока.

Для каждого проводника — твер-дого, жидкого и газообразного — существует определенная зависи-мость силы тока от приложенной разности потенциалов на концах про-водника. Эту зависимость выражает так называемая вольт — амперная ха-рактеристика проводника. Ее нахо-дят, измеряя силу тока в проводнике при различных значениях напряже-ния. Знание вольт — амперной характе-ристики играет большую роль при изучении электрического тока.

Закон Ома.

Наиболее простой вид имеет вольт — амперная характеристи-ка металлических проводников и растворов электролитов. Впервые (для металлов) ее установил немец-кий ученый Георг Ом, поэтому зависимость силы тока от напря-жения носит название закона Ома. На участке цепи, изображенной на рисунке 109, ток направлен от точки 1 к точке 2. Разность потен-циалов (напряжение) на концах проводника равна: U = φ 1 — φ 2. Так как ток направлен слева направо, то напряженность электрического поля направлена в ту же сторону и φ 1 > φ 2

Согласно закону Ома для участка цепи сила тока прямо пропорцио-нальна приложенному напряжению U и обратно пропорциональна сопро-тивлению проводника R:

Закон Ома имеет очень простую форму, но доказать эксперименталь-но его справедливость довольно трудно. Дело в том, что разность по-тенциалов на участке металлическо-го проводника даже при большой силе тока мала, так как мало сопро-тивление проводника.

Электрометр, о котором шла речь, непригоден для измерения столь малых напряжений: его чув-ствительность слишком мала. Нужен несравненно более чувствительный прибор. Тогда, измеряя силу тока амперметром, а напряжение чув-ствительным электрометром, можно убедиться в том, что сила тока пря-мо пропорциональна напряжению. Применение же обычных приборов для измерения напряжения — вольт-метров — основано на использовании закона Ома.

Принцип устройства, вольтметра такой же, как и ампер-метра. Угол поворота стрелки прибо-ра пропорционален силе тока. Сила тока, проходящего по вольтметру, определяется напряжением между точками цепи, к которой он под-ключен. Поэтому, зная сопротивле-ние вольтметра, можно по силе тока определить напряжение. На практике прибор градуируют так, чтобы он сразу показывал напряжение в воль-тах.

Сопротивление. Основная элек-трическая характеристика проводни-ка — сопротивление. От этой вели-чины зависит сила тока в провод-нике при заданном напряжении. Со-противление проводника представля-ет собой как бы меру противо-действия проводника установлению в нем электрического тока. С помощью закона Ома можно определить сопротивление проводника:

Для этого нужно измерить напря-жение и силу тока.

Сопротивление зависит от мате-риала проводника и его геометри-ческих размеров. Сопротивление про-водника длиной l с постоянной пло-щадью поперечного сечения S равно:

где р — величина, зависящая от рода вещества и его состояния (от тем-пературы в первую очередь). Вели-чину р называют удельным сопро-тивлением проводника. Удельное со-противление численно равно сопро-тивлению проводника, имеющего форму куба с ребром 1 м, если ток направлен вдоль нормали к двум противоположным граням куба.

Единицу сопротивления провод-ника устанавливают на основе зако-на Ома и называют ее ом. Провод-ник имеет сопротивление 1 Ом, если при разности потенциалов 1 В сила тока в нем 1 А.

Единицей удельного сопротивле-ния является 1 Ом?м. Удельное со-противление металлов мало. Диэлектрики обладают очень большим удельным сопротивлением. В табли-це на форзаце приведены примеры значений удельного сопротивления некоторых веществ.

Значение закона Ома.

Закон Ома определяет силу тока в электриче-ской цепи при заданном напря-жении и известном сопротивлении. Он позволяет рассчитать тепловые, химические и магнитные действия тока, так как они зависят от силы тока. Из закона Ома вытекает, что замыкать обычную осветительную сеть проводником малого сопротив-ления опасно. Сила тока окажется настолько большой, что это может иметь тяжелые последствия.

Закон Ома — основа всей элект-ротехники постоянных токов. Формулу — надо хорошо понять и твердо запомнить.


ЭЛЕКТРИЧЕСКИЕ ЦЕПИ. ПОСЛЕДОВАТЕЛЬНОЕ И ПАРАЛЛЕЛЬНОЕ СОЕДИНЕНИЯ ПРОВОДНИКОВ

От источника тока энергия может быть передана по проводам к устрой-ствам, потребляющим энергию: Элек-трической лампе, радиоприемнику и др. Для этого составляют электри-ческие цепи различной сложности. Электрическая цепь состоит из источника энергии, устройств, по-требляющих электрическую энергию, соединительных проводов и выклю-чателей для замыкания цепи. Часто и электрическую цепь включают приборы, контролирующие силу тока и напряжение на различных участ-ках цепи, — амперметры и вольт-метры.

К наиболее простым и часто встречающимся соединениям провод-ников относятся последовательное и параллельное соединения.

Последовательное соединение проводников.

При последовательном соединении электрическая цепь не имеет разветвлений. Все проводники включают в цепь поочередно друг за другом. На рисунке 1 показано последовательное соединение двух проводников 1 и 2, имеющих сопротивления R 1 , и R 2 . Это могут быть две лампы, две обмотки элект-родвигателя и др.

Сила тока в обоих проводниках одинакова, т. е. (1)

так как в проводниках электриче-ский заряд в случае постоянного тока не накапливается и через любое поперечное сечение проводника за определенное время проходит один и тот же заряд.

Напряжение на концах рассмат-риваемого участка цепи складывает-ся из напряжений на — первом и вто-ром проводниках:

Надо надеяться, что с доказатель-ством этого простого соотношения вы справитесь сами.

Применяя закон Ома для всего участка в целом и для участков с сопротивлениями R 1 и R 2 , можно до-казать, что полное сопротивление всего участка цепи при последова-тельном соединении равно:

Это правило можно применить для любого числа последовательно соединенных проводников.

Напряжения на проводниках и их сопротивления при последователь-ном соединении связаны соотноше-нием:

Докажите это равенство.

Параллельное соединение про-водников.

На рисунке 2 показано параллельное соединение двух про-водников 1 и 2с сопротивлениями R 1 и R 2 . В этом случае электриче-ский ток 1 разветвляется на две час-ти. Силу тока в первом и втором про-водниках обозначим через I 1 и I 2 . Так как в точке а — разветвлении проводников (такую точку называют узлом) — электрический заряд не на-капливается, то заряд, поступающий в единицу времени в узел, равен заряду, уходящему из узла за это же время. Следовательно, I = I 1 + I 2

Напряжение U на концах про-водников, соединенных параллельно, одно и то же.

В осветительной сети поддержи-вается напряжение 220 или 127 В. На это напряжение рассчитаны при-боры, потребляющие электрическую энергию. Поэтому параллельное сое-динение — самый распространенный способ соединения различных потре-бителей. В этом случае выход из строя одного прибора не отражается на работе остальных, тогда как при последовательном соединении выход из строя одного прибора размы-кает цепь.

Применяя закон Ома для всего участка в целом и для участков с сопротивлениями R 1 и R 2 , можно доказать, что величина, обратная полному сопротивлению участка ab, равна сумме величин, обратных сопротивлениям отдельных провод-ников:

Сила тока в каждом из провод-ников и сопротивления проводников при параллельном соединении свя-заны соотношением

Различные проводники в цепи соединяются друг с другом после-довательно или параллельно. В пер-вом случае сила тока одинакова во всех проводниках, а во втором слу-чае одинаковы напряжения на про-водниках. Чаще всего к осветитель-ной сети различные потребители тока подключаются параллельно.

ИЗМЕРЕНИЕ СИЛЫ ТОКА И НАПРЯЖЕНИЯ

Как измерить силу тока ампер-метром, а напряжение вольтметром, должен знать каждый.

Измерение силы тока.

Для изме-рения силы тока в проводнике ам-перметр включают последовательно с этим проводником (рис. 1). Но нужно иметь в виду, что сам ампер-метр обладает некоторым сопротив-лением R a . Поэтому сопротивление участка цепи с включенным ампер-метром увеличивается, и при неиз-менном напряжении сила тока умень-шается в соответствии с законом Ома. Чтобы амперметр оказывал как можно меньшее влияние на силу тока, измеряемую им, его сопротив-ление делают очень малым. Это нужно помнить и никогда не пытать-ся измерять силу тока в освети-тельной сети, подключая амперметр к розетке. Произойдет короткое за-мыкание; сила тока при малом со-противлении прибора достигнет столь большой величины, что обмотка ам-перметра сгорит.

Измерение напряжения.

Для того чтобы измерить напряжение на участке цепи с сопротивлением R, к нему параллельно подключают вольтметр. Напряжение на вольтметре совпа-дает с напряжением на участке цепи (рис. 2).

Если сопротивление вольтметра R B , то после включения его в цепь сопротивление участка будет уже не R, а . Из-за этого измеряемое напряжение на участ-ке цепи уменьшится. Для того чтобы вольтметр не вносил заметных иска-жений в измеряемое напряжение, его сопротивление должно быть большим по сравнению с сопротивлением участка цепи, на котором измеряется напряжение. Вольтметр можно вклю-чать в сеть без риска, что он сгорит, если только он рассчитан на напря-жение, превышающее напряжение сети.

Амперметр включают последова-тельно с проводником, в котором измеряют силу тока. Вольтметр включают параллельно проводнику, на котором измеряют напряжение.

РАБОТА И МОЩНОСТЬ ПОСТОЯННОГО ТОКА

Электрический ток получил такое широкое применение потому, что он несет с собой энергию. Эта энергия может быть превращена в любую форму.

При упорядоченном движении за-ряженных частиц в проводнике электрическое поле совершает ра-боту; ее принято называть работой тока. Сейчас мы напомним сведения о работе и мощности тока из курса физики VIII класса.

Работа тока.

Рассмотрим произ-вольный участок цепи. Это, может быть однородный проводник, напри-мер нить лампы накаливания, обмот-ка электродвигателя и др. Пусть за время t через поперечное сечение проводника проходит заряд q. Тогда электрическое поле совершит работу A = qU.

Так как сила тока , то эта работа равна:

Работа тока на участке цепи равна произведению силы тока, на-пряжения и времени, в течение ко-торого совершалась работа.

Согласно закону сохранения энергии эта работа должна быть рав-на изменению энергии рассматри-ваемого участка цепи. Поэтому энер-гия, выделяемая на данном участке цепи за время At, равна работе тока (см. формулу (1)).

В случае если на участке цепи не совершается механическая рабо-та и ток не производит химических действий, происходит только нагре-вание проводника. Нагретый про-водник отдает теплоту окружающим телам.

Нагревание проводника происхо-дит следующим образом. Электриче-ское поле ускоряет электроны. После столкновения с ионами кристалличе-ской решетки они передают ионам свою энергию. В результате энергия беспорядочного движения ионов око-ло положений равновесия возраста-ет. Это и означает увеличение внут-ренней энергии. Температура про-водника при этом повышается, и он начинает передавать теплоту окру-жающим телам. Спустя небольшое время после замыкания цепи процесс устанавливается, и температура пе-рестает изменяться со временем. К проводчику за счет работы элект-рического поля непрерывно поступа-ет энергия. Но его внутренняя энер-гия остается неизменной, так как проводник передает окружающим те-лам количество теплоты, равное ра-боте тока. Таким образом, формула (1) для работы тока определяет количество теплоты, передаваемое проводником другим телам.

Если в формуле (1) выразить либо напряжение через силу тока, либо силу тока через напряжение с помощью закона Ома для участка цепи, то получим три эквивалентные формулы:

(2)

Формулой A = I 2 R t удобно пользоваться для последовательного соединения проводников, так как сила тока в этом случае одинакова во всех проводниках. При парал-лельном соединении удобна формула , так как напряжение на всех проводниках одинаково.

Закон Джоуля — Ленца.

Закон, определяющий количество теплоты, которое выделяет проводник с то-ком в окружающую среду, был впервые установлен эксперименталь-но английским ученым Д. Джоу-лем (1818-1889) и русским ученым Э. X. Ленцем (1804-1865). Закон Джоуля — Ленца был сформулиро-ван следующим образом: количество теплоты, выделяемое проводником с током, равно произведению квад-рата силы тока, сопротивления про-водника и времени прохождения то-ка по проводнику:

(3)

Мы получили этот закон с по-мощью рассуждений, основанных на законе сохранения энергии. Формула (3) позволяет вычислить количе-ство теплоты, выделяемое на любом участке цепи, содержащем какие угодно проводники.

Мощность тока.

Любой электри-ческий прибор (лампа, электродвигатель) рассчитан на потребление определенной энергии в единицу вре-мени. Поэтому наряду с работой то-ка очень важное значение имеет по-нятие мощность тока. Мощность то-ка равна отношению работы тока за время t к этому интервалу времени.

Согласно этому определению

(4)

Это выражение для мощности можно переписать в нескольких эквивалентных формах, если исполь-зовать закон Ома для участка цепи:

На большинстве приборов ука-зана потребляемая ими мощность.

Прохождение по проводнику электрического тока сопровождается выделением в нем энергии. Эта энер-гия определяется работой тока: про-изведением перенесенного заряда и напряжения на концах проводника.

ЭЛЕКТРОДВИЖУЩАЯ СИЛА.

Любой источник тока характеризуется электродвижущей силой, или ЭДС. Так, на круглой батарейке для карманного фонарика написано: 1,5 В. Что это значит?

Соедините проводником два ме-таллических шарика, несущих за-ряды противоположных знаков. Под влиянием электрического поля этих зарядов в проводнике возникает электрический ток (рис. 1). Но этот ток будет очень кратковремен-ным. Заряды быстро нейтрализуют-ся, потенциалы шариков станут одинаковыми, и электрическое поле ис-чезнет.

Сторонние силы.

Для того чтобы ток был постоянным, надо поддер-живать постоянное напряжение меж-ду шариками. Для этого необходимо устройство (источник тока), которое перемещало бы заряды от одного шарика к другому в направлении, противоположном направлению сил, действующих на эти заряды со сто-роны электрического поля шариков. В таком устройстве на заряды, кро-ме электрических сил, должны дей-ствовать силы не электростатического происхождения (рис. 2). Одно лишь электрическое поле заряжен-ных частиц (кулоновское поле) не способно поддерживать постоянный ток в цепи.

Любые силы, действующие на электрически заряженные частицы, за исключением сил электростати-ческого происхождения (т. е. кулоновских), называют сторонними си-лами.

Вывод о необходимости сторон-них сил для поддержания посто-янного тока в цепи станет еще оче-виднее, если обратиться к закону сохранения энергии. Электростатиче-ское поле потенциально. Работа это-го поля при перемещении заряжен-ных частиц вдоль замкнутой электри-ческой цепи равна нулю. Прохож-дение же тока по проводникам сопровождается выделением энер-гии — проводник нагревается. Сле-довательно, в любой цепи должен быть какой-то источник энергии, по-ставляющий ее в цепь. В нем, по-мимо кулоновских сил, обязательно должны действовать сторонние не- потенциальные силы. Работа этих сил вдоль замкнутого контура долж-на быть отлична от нуля. Именно в процессе совершения работы этими силами заряженные частицы приобретают внутри источника тока энер-гию и отдают ее затем проводникам электрической цепи.

Сторонние силы приводят в дви-жение заряженные частицы внутри всех источников тока: в генераторах на электростанциях, в гальваниче-ских элементах, аккумуляторах и т.д.

При замыкании цепи создается электрическое поле во всех провод-никах цепи. Внутри источника тока заряды движутся под действием сторонних сил против кулоновских сил (электроны от положительно заряженного электрода к отрица-тельному), а во всей остальной цепи их приводит в движение электриче-ское поле (см. рис. 2).

Аналогия между электрическим током и течением жидкости.

Чтобы лучше понять механизм возникнове-ния тока, обратимся к сходству меж-ду электрическим током в провод-нике и течением жидкости по трубам.

На любом участке горизонталь-ной трубы жидкость течет за счет разности давлений на концах участ-ка. Жидкость перемещается в сторо-ну уменьшения давления. Но сила давления в жидкости — это вид сил упругости, которые являются потен-циальными, подобно кулоновским силам. Поэтому работа этих сил на замкнутом пути равна нулю и одни эти силы не способны вызвать длительную циркуляцию жидкости по трубам. Течение жидкости сопро-вождается потерями энергии вслед-ствие действия сил трения. Для цир-куляции воды необходим насос.

Поршень этого насоса действует на частички жидкости и создает по-стоянную разность давлений на вхо-де и выходе насоса (рис. 3). Благодаря этому жидкость течет по трубе. Насос подобен источнику тока, а роль сторонних сил играет сила, действующая на воду со стороны движущегося поршня. Внутри на-соса жидкость течет от участков с меньшим давлением к участкам с большим давлением. Разность дав-лений аналогична напряжению.

Природа сторонних сил.

Природа сторонних сил может быть разнооб-разной. В генераторах электростанций сторонняя сила — это сила, дей-ствующая со стороны магнитного поля на электроны в движущемся проводнике. Об этом кратко гово-рилось в курсе физики VIII класса.

В гальваническом элементе, на-пример элементе Вольта, действуют химические силы. Элемент Вольта состоит из цинкового и медного электродов, помещенных в раствор серной кислоты. Химические силы вызывают растворение цинка в кис-лоте. В раствор переходят положи-тельно заряженные ионы цинка, а сам цинковый электрод при этом заряжается отрицательно. (Медь очень мало растворяется в серной — кислоте.) Между цинковым и мед-ным электродами появляется раз-ность потенциалов, которая обуслов-ливает ток в замкнутой электриче-ской цепи.

Электродвижущая сила.

Дейст-вие сторонних сил характеризуется важной физической величиной, на-зываемой электродвижущей силой (сокращенно ЭДС).

Электродви-жущая сила в замкнутом контуре представляет собой отношение рабо-ты сторонних сил при перемещении заряда вдоль контура к заряду:

Электродвижущую силу выража-ют в вольтах.

Можно говорить об электродви-жущей силе на любом участке цепи. Это удельная работа сторонних сил (работа по перемещению единич-ного заряда) не во всем контуре, а только на данном участке. Электро-движущая сила гальванического эле-мента есть работа сторонних сил при перемещении единичного положи-тельного заряда внутри элемента от одного полюса к другому. Работа сторонних сил не может быть вы-ражена через разность потенциалов, так как сторонние силы не потенциальные и их работа зависит от формы траектории. Так, например, работа сторонних сил при переме-щении заряда между клеммами ис-точника тока вне самого источника равна нулю.

Теперь вы знаете, что такое ЭДС. Если на батарейке написано 1,5 В, то это означает, что сторонние силы (химические в данном случае) совер-шают работу 1,5 Дж при переме-щении заряда в 1 Кл от одного полюса батарейки к другому. Постоянный ток не может существовать в замкнутой цепи, если в ней не действуют сторонние силы, т. е. нет ЭДС

Рис №1 Рис №2 Рис №3

ЗАКОН ОМА ДЛЯ ПОЛНОЙ ЦЕПИ

Электродвижущая сила опреде-ляет силу тока в замкнутой электри-ческой цепи с известным сопротив-лением.

Спомощью закона сохранения энергии найдем зависимость силы тока от ЭДС и сопротивления.

Рассмотрим простейшую полную (замкнутую) цепь, состоящую из источника тока (гальванического элемента, аккумулятора или гене-ратора) и резистора сопротивле-нием R (рис. 1). Источник тока имеет ЭДС εи сопротивление r. Сопротивление источника часто на-зывают внутренним сопротивлением в отличие от внешнего сопротивле-ния R цепи. В генераторе r — это сопротивление обмоток, а в гальва-ническом элементе — сопротивление раствора электролита и электродов.

Закон Ома для замкнутой цепи связывает силу тока в цепи, ЭДС и полное сопротивление R + r цепи. Эта связь может быть установлена теоретически, если использовать за-кон сохранения энергии и закон Джоуля — Ленца.

Пусть за время t через попе-речное сечение проводника пройдет электрический заряд q. Тогда рабо-ту сторонних сил при перемещении заряда?qможно записать так: А ст = ε · q. Согласно определению силы тока q = It. Поэтому

(1)

При совершении этой работы на внутреннем и внешнем участках цепи, сопротивления которых r и R, выделяется некоторое количество теплоты. По закону Джоуля — Лен-ца оно равно:

Q = I 2 R · t + I 2 r · t. (2)

Согласно закону сохранения энергии A = Q. Приравнивая (1) и (2), получим:

ε = IR + Ir (3)

Произведение силы тока и сопро-тивления участка цепи часто назы-вают падением напряжения на этом участке. Таким образом, ЭДС равна сумме падений напряжений на внут-реннем и внешнем участках замкну-той цепи.

Обычно закон Ома для замкну-той цепи записывают в форме

(4)

Инструкция

Видео по теме

Компаратор как замена конденсатору в обычной игре

В обычном (без плагинов и модов) варианте Minecraft такого понятия, как конденсатор, не существует. Вернее, устройство, выполняющее его функции, имеется, но название у него совершенно другое — компаратор. Некоторая путаница в этом плане произошла еще в период разработки такого прибора. Сперва — в ноябре 2012-го — представители Mojang (компании-создателя игры) объявили о скором появлении в геймплее конденсатора. Однако через месяц они высказались уже о том, что как такового этого прибора не будет, а вместо него в игре будет компаратор.

Подобное устройство существует для проверки заполненности расположенных позади него контейнеров. Таковыми могут быть сундуки (в том числе в виде ловушек), варочные стойки, раздатчики, выбрасыватели, печи, загрузочные воронки и т.п.

Помимо этого, его часто используют для сравнения двух сигналов редстоуна между собою — он выдает результат в соответствии с тем, как было запрограммировано в данной цепи, и с тем, какой режим выбран для самого механизма. В частности, компаратор может разрешить зажигание факела, если первый сигнал больше либо равен другому.

Также порой конденсатор-компаратор устанавливают рядом с проигрывателем, подключая его входом к последнему. Когда в звуковоспроизводящем устройстве проигрывается какая-либо пластинка, вышеупомянутый прибор будет выдавать сигнал, равный по силе порядковому номеру диска.

Скрафтить такой компаратор несложно, если имеется достаточно трудно добываемый ресурс — адский . Его надо поставить в центральный слот верстака, над ним и по бокам от него установить три красных факела, а в нижнем ряду — такое же количество каменных блоков.

Конденсаторы, встречающиеся в разных модах Minecraft

В большом количестве модов попадаются конденсаторы, имеющие самое разное предназначение. К примеру, в Galacticraft, где у геймеров есть возможность слетать на многие планеты для ознакомления с тамошними реалиями, появляется рецепт крафта кислородного конденсатора. Он служит для создания механизмов вроде коллектора и накопителя газа для , а также рамки воздушного шлюза. Для его изготовления четыре стальных пластины размещаются по углам верстака, в центре — оловянная канистра, а под нею — воздуховод. Остальные три ячейки занимают пластины из олова.

В JurassiCraft существует конденсатор потока — некий телепорт, позволяющий переместиться в удивительный игровой мир, кишащий динозаврами. Для создания такого прибора нужно поместить в два крайних вертикальных ряда шесть железных слитков, а в средний — два алмаза и между ними единицу пыли редстоуна. Дабы устройство заработало, надо поставить его на свинью либо вагонетку, а затем щелкнуть по нему правой клавишей мыши, быстро запрыгнув туда. При этом требуется поддержание высокой скорости устройства.

С модом Industrial Craft2 у игрока появляется возможность создавать как минимум два вида тепловых конденсаторов — красный и лазуритовый. Они служат исключительно для охлаждения ядерного реактора и для накопления его энергии и хороши для циклических сооружений такого типа. Остужаются они сами, соответственно, красной пылью или лазуритом.

Красный теплоконденсатор делается из семи единиц пыли редстоуна — их надо установить в виде буквы П и расставить под ними теплоотвод и теплообменник. Крафтинг же лазуритового устройства чуть посложнее. Для его создания четыре единицы пыли редстоуна расставляются по углам станка, в центр пойдет блок лазурита, по бокам от него — два красных тепловых конденсатора, сверху — теплоотвод реактора, а снизу — его же теплообменник.

В ThaumCraft, где сделан акцент на настоящем чародействе, конденсаторы тоже используются. Например, один из них — кристаллический — существует для аккумуляции и отдачи магии. Причем, что интересно, создавать его и многие другие вещи разрешено лишь после изучения особого элемента геймплея — исследования, проводимого за специальным столом и с определенными приборами.

Делается такой конденсатор из восьми тусклых осколков, в центр которых на верстаке помещается мистический деревянный блок. К сожалению, подобный прибор — равно как и его составляющие — просуществовал лишь до ThaumCraft 3, а в четвертой версии мода был упразднен.

Источники:

  • О компараторе в Minecraft
  • Кислородный конденсатор в Galacticraft
  • Мод JurassiCraft
  • Ядерный реактор в Industrial Craft2
  • Кристаллический конденсатор в ThaumCraft

Facebook

Twitter

Вконтакте

Одноклассники

Google+

Задачи на нахождение силы тока в цепи, напряжения. Как решать задачи на закон Ома.

Задачи на силу тока в основном касаются определения силы тока, напряжения и сопротивления. В данном разделе Вы найдете формулы для решения задач. Мы разберем решение типичных элементарных задач, используя закон Ома.

Задача 1. Сила тока

Через нить накаливания лампочки от карманного фонарика за 2 мин проходит электрический заряд, равный 30 Кл. Определите силу тока в этой лампочке.

Дано:    Решение
q = 30 Кл   Сила тока I определяется по формуле
t = 2 мин  120 сI= q/t
I – ?   I = 30Кл/120с = 0,25А = 250м А
Сила тока. Решение задач

Ответ: I = 250 мА

Задача 2. Напряжение в цепи

Электродвигатель включен в электрическую цепь с напряжением 24В. Определите заряд, прошедший через электродвигатель, если при этом была  совершена работа, равная 84 кДж

Дано:    Решение
U = 24 В   Напряжение на электродвигателе определяется по формуле
А = 84 кДж  84000  ДжU=A/q
q= A/U
q – ?    Q = 84000 Дж/24 В = 3500 Кл
Напряжение в цепи. Решение задач

Ответ: q = 3500 Кл

Задача 3. Закон Ома сила тока

Определите силу тока в кипятильнике, включенном в сеть с напряжением 220 В, если сопротивление спирали составляет 55 Ом.

Дано:    Решение
U = 220 В   Силу тока в кипятильнике можно определить, воспользовавшись законом Ома
R = 55 Ом   І=U/R
І – ?    І = 220 B/55 Ом = 4 А
Сила тока по закону Ома. Решение задач

Ответ: І = 4 А

Задача 4. Закон Ома напряжение

Какое напряжение нужно приложить к концам проводника сопротивлением 5 Ом, тобі по проводнику пошел ток с силой тока, равной 300 мА

Дано:    Решение
R = 5 Ом   Необходимое напряжение можно определить, воспользовавшись законом Ома
І = 300 мА0.3 A І=U/R
U = IR  
U – ?    U = 0,3 А * 5 Ом = 1,5 В
Напряжение в сети. Закон Ома

Ответ: U = 1.5 B

Задача 5. Сопротивление по закону Ома

Определите cопротивление резистора, если за время 10 мин через него проходит заряд 200 Кл. Напряжение на концах резистора равно 6 В.

Дано:    Решение
t = 10 мин q = 200 Кл U = 6 В  600 сСопротивление резистора можно определить, воспользовавшись законом Ома
    І=U/R
R=U/I
Поскольку значение силы тока I  не задано по условию задачи, его можно определить по формуле
I=q/t
Подставив формулу силы тока в формулу сопротивления, получим
R=Ut/q
R – ?    R = 6 В *600с/200 Кл = 18 Ом  
Сопротивление по закону Ома

Ответ: R = 18 Ом

определение, формула, физический смысл, применение

Силой Лоренца называют силу, которая действует со стороны электромагнитного поля на движущийся электрический заряд. Весьма нередко силой Лоренца называют лишь магнитную составляющую этого поля. Формула для определения:

F = q(E+vB),

где q — заряд частицы; Е — напряжённость электрического поля; B — магнитная индукция поля; v — скорость частицы. 

Сила Лоренца очень похожа по своему принципу на силу Ампера, разница заключается в том, что последняя действует на весь проводник, который в целом электрически нейтральный, а сила Лоренца описывает влияние электромагнитного поля лишь на единичный движущийся заряд.

Она характеризуется тем, что не изменяет скорость перемещения зарядов, а лишь воздействует на вектор скорости, то есть способна изменять направление движения заряженных частиц.

В природе сила Лоренца позволяет защищать Землю от воздействия космической радиации. Под её воздействием падающие на планету заряженные частицы отклоняются от прямой траектории благодаря присутствию магнитного поля Земли, вызывая полярные сияния.

В технике сила Лоренца используется очень часто: во всех двигателях и генераторах именно она приводит во вращение ротор под действием электромагнитного поля статора.

Таким образом, в любых электромоторах и электроприводах основным видом силы является Лоренцева. Кроме того, она применяется в ускорителях заряженных частиц, а также в электронных пушках, которые раньше устанавливались в ламповых телевизорах. В кинескопе испускаемые пушкой электроны отклоняются под влиянием электромагнитного поля, что происходит при участии Лоренцевой силы.

Кроме того, эта сила используется в масс-спектрометрии и масс-электрографии для приборов, способных сортировать заряженные частицы в зависимости от их удельного заряда (отношение заряда к массе частицы). Это позволяет с высокой точностью определять массу частиц. Также находит применение в других КИП, например, в бесконтактном способе измерения расхода электропроводящих жидких сред (расходомеры). Это очень актуально, если жидкая среда обладает очень высокой температурой (расплав металлов, стекла и др.).

Как рассчитать кулоны | Sciencing

Обновлено 3 ноября 2020 г.

Клэр Гиллеспи

Электрический заряд, который проходит через что угодно, от батареи AA до молнии, измеряется в кулонах. Если вы знаете, какой ток в цепи и как долго он течет, вы можете рассчитать электрический заряд в кулонах.

Свойства кулонов

Электроны крошечные и имеют очень маленький заряд. В физике очень большое количество электронов определяется как 1 единица заряда, называемая кулоном.Один кулон эквивалентен 62 × 10 18 электронам. Количество кулонов в секунду называется током (т. Е. Скоростью потока кулонов в цепи). Энергия кулона называется напряжением и измеряется в джоулях.

Как рассчитать электрический заряд

Чтобы определить величину электрического заряда, протекающего в цепи, вам необходимо знать, какой ток течет и как долго он протекает. Уравнение:

\ text {заряд в кулонах} = \ text {ток в амперах} \ times \ text {время в секундах}

Например, если ток 20 А течет в течение 40 с, расчет равен 20. × 40.Таким образом, электрический заряд равен 800 C.

Как рассчитать переданную энергию

Если вы знаете количество электрического заряда в кулонах и напряжение (также известное как разность потенциалов), вы можете вычислить, сколько энергии передается. Уравнение:

\ text {энергия, преобразованная в джоулях} = \ text {разность потенциалов в вольтах} \ times \ text {заряд в кулонах}

Например, если разность потенциалов составляет 100 В, а заряд равен 3 Кл. , расчет 100 × 3.Таким образом передается 300 Дж энергии.

Использование закона Кулона

Произведение электрических зарядов в двух телах (т. Е. Притягиваются они или отталкиваются друг от друга) зависит от заряда каждого тела в кулонах, а также от расстояния между телами. Если полярности одинаковы (обе положительные или обе отрицательные), кулоновская сила отталкивается, но если полярности противоположны (отрицательная / положительная или положительная / отрицательная), кулоновская сила притягивается. Электрический заряд также обратно пропорционален квадрату расстояния между двумя телами.Это известно как закон Кулона, который сформулирован как:

В этом уравнении F — сила, приложенная к зарядам (q 1 ) и (q 2 ), k — постоянная Кулона и (r) — расстояние между (q 1 ) и (q 2 ). Значение k зависит от среды, в которую погружены заряженные объекты. Например, значение воздуха составляет приблизительно 9,0 × 10 9 Нм 2 / C 2 . Закон Кулона можно использовать для решения многих физических задач, в которых известны все значения, кроме одного.

Электрический заряд (Q)

Что такое электрический заряд?

Электрический заряд генерирует электрическое поле. Электрический заряд влияет на другие электрические заряды с электрической силой и под влиянием другого заряжается с той же силой в противоположном направлении.

Есть 2 вида электрического заряда:

Положительный заряд (+)

Положительный заряд имеет больше протонов, чем электронов (Np> Ne).

Положительный заряд обозначается знаком плюс (+).

Положительный заряд притягивает другие отрицательные заряды и отталкивает другие положительные заряды.

Положительный заряд притягивается другим отрицательным заряды и отражаются другими положительными зарядами.

Отрицательный заряд (-)

Отрицательный заряд содержит больше электронов, чем протонов (Ne> Np).

Отрицательный заряд обозначается знаком минус (-).

Отрицательный заряд притягивает другие положительные заряды и отталкивает другие отрицательные заряды.

Отрицательный заряд притягивается другим положительным заряды и отталкиваются другими отрицательными зарядами.

Направление электрической силы (F) в зависимости от типа заряда
начисления 1/2 кв. Усилие на q 1 Заряд Усилие на q 2 Заряд
— / — ← ⊝ ⊝ → пополнение
+ / + ← ⊕ ⊕ → пополнение
— / + ⊝ → ← ⊕ аттракцион
+ / — ⊕ → ← ⊝ аттракцион
Заряд элементарных частиц
Частица Заряд (К) Заряд (е)
Электрон 1.602 × 10 -19 С

e

Протон 1,602 × 10 -19 С

+ e

Нейтрон 0 С 0

Кулон

Электрический заряд измеряется в кулонах [Кл].

Один кулон имеет заряд 6,242 × 10 18 электроны:

1C = 6,242 × 10 18 e

Электрический заряд расчет

Когда электрический ток течет в течение определенного времени, мы можем рассчитать сбор:

Постоянный ток

Q = I т

Q — электрический заряд, измеренный в кулоны [C].

I — ток, измеренный в амперах. [А].

t — период времени, измеряемый в секунды [с].

Кратковременный ток

Q — электрический заряд, измеренный в кулоны [C].

i ( t ) — мгновенный ток, измеряется в амперах [A].

t — период времени, измеряемый в секунды [с].


См. Также

123 страницы по физике Стива Боддекера;

Ch31 Электрический ток / постоянный ток Схемы

Мы обсуждаем электрический ток с тех пор, как первый день нашего класса

I имеет единицы Ампера = кулон в секунду

I = кв / т

Электрический ток

В батарее используются химические реакции для создания разности потенциалов между его выводами при отключении от цепи.

EMF Странный термин, но давайте относимся к вашему первому класс механики.

Объем работы, необходимый для перемещения заряда ΔQ от одного вывода к другому — ЭДС, ε

Работа = ΔQ ε

Если вы толкнете предмет по полу с усилие 100 Н через 10 метров, сколько работы вы сделали?

Работа = F Δx

Работа = 100 Н (10 м)

Работа = 1000 Нм или 1000 Дж

Мы знаем из P = I V, что вольт имеет единицы измерения энергии на единицу заряда.

Резюме

п = I V

W = A (В) (единицы)

Дж / с = C / с (В)

Вольт = Дж / К

Итак, если у нас есть замкнутый цикл и заряд, q, завершает весь цикл, сила, которая заставила этот заряд переместить на всей длине пути это можно описать как F Δx на единицу заряда.

Мы также знаем, что Нм / Кл — это единицы напряжения, поэтому ЭДС также имеет единицы измерения напряжения.

Почему физики говорят, что заряд перетекает с положительного на отрицательный?

The направление тока от положительного вывода к отрицательному было решено до того, как было осознано, что электроны заряжены отрицательно.

Теперь с этим сказанным никогда не говорите текущий расход, так как ток является расходом заряда

Текущий расход

Расход наддува

Примечание: реальное движение электронов по проводу вполне медленный; электроны проводят большую часть своего времени, беспорядочно подпрыгивая, и имеют лишь небольшую составляющую скорости, противоположную направлению заряда поток.

Пример

Литий-йодные батареи

рассчитаны на одиночные заряд, долгосрочное развертывание. Если у одной из этих батарей 1,0 А-ч накопленный заряд, сколько кулонов хранится в этой батарее? Если это батарея потребляет 10 мкА, каков ожидаемый срок службы этого аккумулятор (в месяцах или годах)?

Q = 1 А-час

Q = 1 Кл / с (1 час)

Q = 1 К / с (3600 с)

Q = 3600 кулонов

I = Q / т

10e-6 = 3600С / т

т = 3.6e8 секунд

т = 3,6e8 с (1ч / 3600с) (1д / 24ч) (1г / 365дн)

t = 11,4 года

Сопротивление и закон Ома

V = I Закон Ромса не является универсальным законом. Этот закон применяется к большинству металлические проводники

Как вы знаете, мне нравится использовать воду для сравнений между током воды и электрическим током. Подобно потенциалам.

В ограниченном диапазоне температур, сопротивления проводников можно округлить до

R = R или (1 αΔT)

Удельное сопротивление это мера проводника, которая не зависит от физического размера провода (длина или площадь)

R = ρ (длина / площадь)

Пример

Транскраниальная стимуляция постоянным током — это лечение используется при тяжелой депрессии, а также при невропатической боли после спинномозговой травмы пуповины.Если через мозг пациента с разность потенциалов между электродами 10,0 В, какова эквивалентное сопротивление между электродами?

Примечание: Не путайте электросудорожную терапию (ЭСТ) и tDCS. ECT (производит судороги) проводится в больнице с побочными эффектами, такими как потеря памяти. tDCS (стимулирует части мозга) выполняется в кабинете врача. Оба используются при депрессии.

В = I R

10 = 0.001 (R)

R = 10,000 Ом

Энергия и Мощность в электрических цепях

Мы знаем потенциальную энергию изменения заряда поскольку он проходит через разность потенциалов.

ΔU = q ΔV

Еще помним Сила = Работа / время

Или

P = ΔU / время

P = q ΔV / время I = q / время

P = I V

(Примечание: помните отрицательный знак назначается только через определения, Работа = -ΔU, но я произвольно изменяя определение этой презентации.)

Ваша книга любит комбинировать уравнения типа

P = I 2 R и P = V 2 / Р; Я НИКОГДА не заморачиваюсь с этими уравнениями

Пример

1 кВт / ч равно сколько Джоулей?

Ответ:

E = 1 кВт-час

E = 1000 Дж / с (1 час)

E = 1000 Дж / с (3600 с)

E = 3.6 e 6 Джоулей

Пример

Генератор переменного тока 110 В обеспечивает мощность 3,3 кВт и имеет КПД 90%. Какой ток вырабатывает генератор?

P = I V

3 300 = I (110)

I = 30A

(Примечание: Мы игнорируем эффективность, это будет использоваться только для определения количества топлива. требуется.)

Резисторы и последовательный и параллельный

серии

Мы знаем, что падение потенциала на каждом резистор должен быть суммирован с полной разностью потенциалов на блоке питания. или

ΣV Итого = V 1 + V 2 + V

Мы также знаем, что заряд сохраняется, поэтому последовательно,

Я = Я 1 = I 2 = I

Разумеется

ΣV Всего = V 1 + V 2 + V

ΣIR Всего = I 1 R 1 + I 2 R 2 + I R

отменить из Is

R Серия = R 1 + R 2 + R

Параллельный

Поскольку заряд сохраняется, мы знаем

I параллельно = I 1 + I 2 + I и т. Д.

Примечание: этот рисунок подразумевает, что сопротивление через R 1 , R 2 и R 3 составляет очень похожий.Если они идентичны, это означает, что i 1 = i 2 = i 3 = I / 3

Пример

Что такое I 1 , I 2 и I 3 ?

1 / R = 1/ 1 + 1/ 2 + 1/ 3

1 / R = 1/3 + 1/4 + 1/5

1 / R = 20/60 + 15/60 + 12/60

R = 60/47 = 1,277 Ом

В = I R

12 = Я 1.277

I = 9,4 А

(20 + 15 + 12 = 47, нормализовано)

I 1 = 20/47 (9,4 A) = 4 A

I 2 = 15/47 (9,4 А) = 3 А

I 3 = 12/47 (9,4 A) = 2,4 A

Пример

R 1 = 10 Ом

R 2 = 6 Ом

R 3 = R 4 = R 5 = 12 Ом

R 6 = 8 Ом

Что ток через каждый резистор?

Сначала нужно найти эквивалентное сопротивление R 3 , 4 и 5 .

1 / R = 1/ 3 + 1/ 4 + 1/ 5

1 / R = 1/12 + 1/12 + 1/12

1 / R = 3/12

R экв-3,4,5 = 12/3 = 4 Ом

Теперь нам нужно определить эквивалентное сопротивление между R 3,4,5 и 6 .

R серии = 3,4,5 + 6

R 3,4,5 -> 6 = 4 + 8 = 12 Ом

У нас новая проблема !!!

R 1 = 10 Ом

R 2 = 6 Ом

R экв = 12 Ом

Оф Конечно, большую часть заряда собираюсь выбрать 2 рэнд на эта точка.

1 / R = 1 / R 1 + 1 / R 2 + 1 / R экв.

1 / R = 1/10 + 1/6 + 1/12

1 / R = 6/60 + 10/60 + 5/60

R = 60/21 = 2,857 Ом

В = I R

12 = I 2,857

I = 4,2 А

I 1 = 4,2 A (6/21) = 1,2 A

I 2 = 4,2 A (10/21) = 2,0 A

I экв = Я 6 = 4.2А (5/21) = 1,0 А

I 3 = I 4 = I 5 = I 6 /3 = 1/3 А

Кирхгоф Правила

Kirchhoffs 1 st Закон

Сохранение электрического заряда дает сумма тока, входящего в любое пересечение или узел, должна равняться сумме любое течение, покидающее перекресток.

на узле

n

ΣI к = 0

к = 1

Аналогично для рек. Если три филиалы встречаются в одной точке,

ветка A при 50 галлонах в секунду,

ветка B при 200 галлонах в секунду

ветка C при 150 галлонах в секунду

Образовавшаяся большая река, выходящая из этого точка должна иметь (50 + 200 + 150) или 400 галлонов в секунду.

Kirchhoffs 1 st Закон гласит, что ток, входящий или выходящий из любого соединения или узла, должен быть сохранен.

i 2 = i 1 + i 3

Пример

E 1 = 10 В

E 2 = 8 В

R 1 = 20 Ом

R 2 = 10 Ом

R 3 = 14 Ом

Kirchhoffs 2 nd Закон

Поскольку энергия должна быть сохранена, мы знаем, что сумма всех напряжений должна быть равна нулю в замкнутом контуре.Это также упоминается как правило Петли Кирхгофа.

n

ΣV к = 0

к = 1

Применить 1 st Закон

Я 2 = Я 1 + Я 3

Применить закон 2 nd

Для верха и

нижние петли

Петля верхняя

E 1 I 1 R 1 Я 2 R 2 = 0

10 20I 1 10I 2 = 0

10 20I 1 10I 1 10I 3 = 0

10 30I 1 10I 3 = 0

снизу Петля

E 2 I 3 R 3 Я 2 R 2 + E 1 = 0

8 14I 3 10I 2 + 10 = 0

18 14I 3 10I 1 10I 3 = 0

18 10I 1 24I 3 = 0

Решите систему уравнений

Умножьте уравнение нижнего контура на коэффициент -3

10 30I 1 10I 3 = 0

-54 + 30I 1 + 72I 3 = 0

62I 3 = 44

Я 3 = 22/31 = 0.71 А

Решить относительно

I 1 10 30I 1 10 (.71) = 0

I 1 = 0,097 А

Цепи Конденсаторы содержащие

Q = C V

Для конденсаторов:

серии: 1 / C экв = 1 / C 1 + 1 / C 2 + + 1 / C N

Параллельный: C экв = C 1 + C 2 + + C N

Вывод мы знаем падение напряжения на каждом конденсаторе, В 1 , В 2 и В 3 суммы к ЭДС источника питания, В

В = В 1 + В 2 + В 3

Q / C = Q / C 1 + Q / C 2 + К / К 3

Ток и заряд в последовательных цепях равны

1 / C экв = 1 / C 1 + 1 / К 2 + 1 / К 3

Вывод мы знаем ток через каждый конденсатор, I 1 , I 2 и I 3 суммы к общему току, I

Я = Я 1 + Я 2 + Я 3

Q = Q 1 + Q 2 + Q 3

В C = V C 1 + V C 2 + V C 3

Разница потенциалов равна параллельные цепи, поэтому V отменяет

C экв = C 1 + С 2 + С 3

Пример

Для серий, измеренная разность потенциалов по C 1 — 5.0 Вольт, где C 1 = 10 F и C 2 = 5 F. Какое падение напряжения около C 2 ?

Пример

Над параллельными конденсаторами емкость равна Учитывая C 1 = 10 Фарад и C 2 = 5 F, что такое C eq ?

Решение

Q сохраняется

Q 1 = Q 2

C 1 V 1 = C 2 В 2

10 (5) = 5 (В 2 )

В 2 = 10 В

Эта концепция необходима для решения последний из 20 баллов поставленных домашних задач

Решение: 5 + 10 = 15 Фарад

RC-схемы

При зарядке

q = C V o (1 — e t / RC )

CV = C V o (1 — e t / RC )

V = V o (1 — e t / RC )

V o (1 — е т / τ )

Так какие единицы измерения RC? Ans Ом (Фарады)

Можно упростить и Ом-Фарад на более основные единицы?

RC Ом (Фарады)

RC В / А (C / V)

RC К / А

RC C / C / с

RC секунд

Текущий

Когда высокий горшок отличается от V или до текущий V

I = (V o / R) e t / RC

I = I o e t / RC

Фотовспышка для фотоаппарата емкостью 5 мФ.Какое сопротивление нужно в этой RC-цепи, если вспышка должна заряжаться до 86,5% от полной зарядки за 10 с?

86,5% V o = V o (1 — e t / RC )

2 = т / (R C)

2 = 10 / (0,005 рэндов)

R = 1000 Ом

Как в части C вашего lab, построить график заданных данных и решить для постоянной времени, τ = RC, и начального разность потенциалов, В не более .

у = м х + Ь

ln (В) = (-1 / τ) t + ln V макс

Если резистор 30 кОм (килоом) и конденсатор 40 мкФ (микрофарады) сколько истекло время достижения трех постоянных времени?

τ = RC = 30e3 Ом * 40e-6F

τ = 1,2 ОмФ или 1,2 секунды

3 (τ) = 3,6 секунды

Какой процент V o остается после истечения трех постоянных времени RC, пока разряжать конденсатор?

V = V o e -t / RC

В

3.00

2,50

2,00

1,50

1,00

0,50

т (сек)

51

76

107

149

208

307

Амперметры и вольтметры

Амперметры измеряют ток и включают последовательно в цепь.Ниже амперметр ставится после первого резистора. Амперметр состоит из гальванометр (с внутренним сопротивлением r) и шунтирующий резистор, позволяет большей части тока обходить гальванометр.

Большинство мультиметров позволяют изменять это значение. шунтирующее сопротивление (с переключателем / тумблером) для измерения в широком диапазоне тока.

Вольтметры измеряют напряжение и включаются параллельно с компонентом измеряется.Ниже вольтметр ставим параллельно 2 nd сопротивляться.

Если через гальванометр (как и амперметр), гальванометр может быть поврежден. Ну и что будет ли величина резистора умножителя обычно?

Ответ: обычно> 1 миллион Ом

Заряд и ток — Заряд, ток и напряжение — CCEA — Редакция GCSE Physics (Single Science) — CCEA

Электрический ток представляет собой поток заряженных частиц.

В металлических проводниках заряженными частицами являются свободные электроны.

Электроны могут свободно перемещаться от одного иона к другому, и чистый поток этих электронов в одном направлении представляет собой электрический ток.

Для движения свободных электронов в одном направлении требуется источник энергии, например элемент или батарея.

Charge

Электроны — это отрицательно заряженные частицы, которые передают электрическую энергию от элемента через проводящие провода в виде электрического тока.

Заряд измеряется в кулонах , C.

Заряд электрона составляет 1,6 x 10 -19 C.

Другими словами, требуется 6,250,000,000,000,000,000 электронов, чтобы составить 1 кулон заряда.

Кулон заряда — это просто очень большая группа электронов.

Связь между током I и количеством заряда Q

Электрический ток — это поток заряженных частиц.

Величина электрического тока — это скорость протекания заряда.

Ток I = \ (\ frac {\ text {количество заряда Q}} {\ text {время t}} \)

I = \ (\ frac {\ text {Q}} {\ text {t }} \)

Это часто называют:

Количество заряда Q = ток I x время t

Q = It

Где:

Q = количество заряда в кулонах, C

I = ток в амперы, А

t = время в секундах, с

I = \ [\ frac {\ text {Q}} {\ text {t}} \] I = Q ÷ t
Q = It Q = I xt
t = \ [\ frac {\ text {Q}} {\ text {I}} \] t = Q ÷ I

Один ампер — это ток, который протекает, когда один кулон заряда проходит через точку в цепи за одну секунду.

Заряд и ток

Заряд и ток
Далее: Напряжение и работа Up: Копаем глубже Предыдущая: Единицы СИ

Частицы с электрическим зарядом действуют друг на друга. Величина этой силы зависит от заряда на каждая частица. Величина силы обратно пропорциональна пропорционально квадрату расстояния между частицы.

Базовой единицей заряда является кулон .Один кулон равен заряду электроны. Другими словами, у одного электрона есть заряд из кулоны. Символ для заряд.

Электрические цепи перемещают электрический заряд так, что полезная работа сделана. Эти движущиеся заряды создают электрический ток , который мы обозначаем как или. В другими словами, если — это сумма заряда на конкретная точка в пространстве во времени, затем текущая проходящая через эту точку равна первой производной по времени из . Другими словами,


Базовая единица тока — ампер (обозначен сокращение).Один ампер равен одному кулону заряд проходит через точку в пространстве за одну секунду. В другими словами, один ампер равен одному кулону в секунду.

Рассмотрим провод, по которому проходит ток в амперы. через определенную точку на этом проводе. Заряд может либо двигаться справа налево, либо слева направо. Так чтобы полностью уточнить природу тока, мы должны также укажите направление, в котором течет ток. путешествия. Это делается путем привязки знака к электрический ток.Другими словами, текущий — это со знаком количество.

Знак, данный току, зависит от того, кем мы являемся. заинтересован в измерении. Движущийся заряд можно представить как либо

  • отрицательно заряженных электронов, движущихся по проводу,
  • или положительно заряженных частиц, движущихся через провод
В первом случае мы имеем так называемый электрон текущий . Во втором случае имеем так называемый условный ток .Обычной практикой является использование обычные, а не электронные токи. На протяжении всего нашего работать, это соглашение, которое мы будем использовать.

На принципиальных схемах мы обозначаем ток, текущий в элемент схемы стрелкой, обозначающей один из терминалы. Стрелка обычно обозначается размером электрический ток. Стандартное соглашение (называемое пассивным соглашение о маркировке), используемое при маркировке этих стрелок, используйте положительное число, когда ток давит положительный заряжается в устройство.Если число отрицательное, то ток вытягивает положительные заряды из устройства. Рисунок 16 иллюстрирует соглашение о пассивной маркировке резистора.

Рисунок 16: Ток, протекающий в элементе цепи

Напомним, что номер, обозначающий текущий, подписан Стрелка. Это означает, что мы можем получить две разные метки для то же направление обычного тока. Фигура 17 показаны две такие метки. В в первом случае мы выталкиваем положительный заряд из терминал в устройство.Во втором случае мы вытягивая положительный заряд из устройства в терминал. Конечный результат для обеих этикеток одинаковый, а именно: поток положительных зарядов идет слева направо через устройство.

Рисунок 17: Две разные маркировки одного и того же тока

Полный заряд, поступающий в элемент схемы, получается следующим образом: интегрируя дифференциальное уравнение. Предположим, что заряд изначально есть, затем общий заряд, поступающий в устройство между временами и будет




Далее: Напряжение и работа Up: Копаем глубже Предыдущая: Единицы СИ
Майкл Леммон 2009-02-01

7.2 Электрический потенциал и разность потенциалов — University Physics Volume 2

Цели обучения

К концу этого раздела вы сможете:

  • Определение электрического потенциала, напряжения и разности потенциалов
  • Определите электрон-вольт
  • Вычислить электрический потенциал и разность потенциалов на основе потенциальной энергии и электрического поля
  • Опишите системы, в которых электрон-вольт является полезной единицей
  • Применение энергосбережения в электрических системах

Напомним, что ранее мы определили электрическое поле как величину, не зависящую от тестового заряда в данной системе, что, тем не менее, позволило бы нам вычислить силу, которая возникнет при произвольном тестовом заряде.(При отсутствии другой информации по умолчанию предполагается, что тестовый заряд положительный.) Мы кратко определили поле для гравитации, но гравитация всегда притягивает, тогда как электрическая сила может быть либо притягивающей, либо отталкивающей. Следовательно, хотя потенциальная энергия вполне достаточна в гравитационной системе, удобно определить величину, которая позволяет нам вычислить работу над зарядом независимо от величины заряда. Непосредственный расчет работы может быть затруднен, поскольку W = F → · d → W = F → · d →, а направление и величина F → F → могут быть сложными для нескольких зарядов, для объектов нечетной формы и вдоль произвольных путей. .Но мы знаем, что, поскольку F → = qE → F → = qE →, работа и, следовательно, ΔU, ΔU, пропорциональны пробному заряду q . Чтобы получить физическую величину, не зависящую от испытательного заряда, мы определяем электрический потенциал В (или просто потенциал, поскольку понимается электрический) как потенциальную энергию на единицу заряда:

Электрический потенциал

Потенциальная электрическая энергия на единицу заряда составляет

Поскольку U пропорционально q , зависимость от q отменяется.Таким образом, V не зависит от q . Изменение потенциальной энергии ΔUΔU имеет решающее значение, поэтому нас беспокоит разность потенциалов или разность потенциалов ΔVΔV между двумя точками, где

ΔV = VB − VA = ΔUq.ΔV = VB − VA = ΔUq.

Разница электрических потенциалов

Разность электрических потенциалов между точками A и B , VB − VA, VB − VA, определяется как изменение потенциальной энергии заряда q , перемещенного с A на B , деленное на заряд.Единицами разности потенциалов являются джоули на кулон, получившие название вольт (В) в честь Алессандро Вольта.

Знакомый термин «напряжение» — это общее название разности электрических потенциалов. Имейте в виду, что всякий раз, когда указывается напряжение, под ним понимается разность потенциалов между двумя точками. Например, каждая батарея имеет две клеммы, а ее напряжение — это разность потенциалов между ними. По сути, точка, которую вы выбираете как ноль вольт, произвольна. Это аналогично тому факту, что гравитационная потенциальная энергия имеет произвольный ноль, например, на уровне моря или, возможно, на полу лекционного зала.Стоит подчеркнуть различие между разностью потенциалов и электрической потенциальной энергией.

Разность потенциалов и электрическая потенциальная энергия

Связь между разностью потенциалов (или напряжением) и электрической потенциальной энергией определяется формулой

. ΔV = ΔUqorΔU = qΔV.ΔV = ΔUqorΔU = qΔV.

7,5

Напряжение — это не то же самое, что энергия. Напряжение — это энергия на единицу заряда. Таким образом, аккумулятор мотоцикла и автомобильный аккумулятор могут иметь одинаковое напряжение (точнее, одинаковую разность потенциалов между выводами аккумулятора), но при этом один хранит гораздо больше энергии, чем другой, потому что ΔU = qΔV.ΔU = qΔV. Автомобильный аккумулятор может заряжать больше, чем аккумулятор мотоцикла, хотя оба аккумулятора — 12 В.

Пример 7,4

Расчет энергии
У вас есть мотоциклетный аккумулятор на 12,0 В, способный обеспечить заряд на 5000 C, и автомобильный аккумулятор на 12,0 В, способный обеспечить заряд на 60 000 C. Сколько энергии дает каждый? (Предположим, что числовое значение каждого заряда имеет точность до трех значащих цифр.)
Стратегия
Сказать, что у нас есть батарея на 12,0 В, означает, что на ее выводах есть 12.Разность потенциалов 0 В. Когда такая батарея перемещает заряд, она пропускает заряд через разность потенциалов 12,0 В, и заряд получает изменение потенциальной энергии, равное ΔU = qΔV.ΔU = qΔV. Чтобы найти выход энергии, мы умножаем перемещенный заряд на разность потенциалов.
Решение
Для аккумуляторной батареи мотоцикла q = 5000Cq = 5000C и ΔV = 12,0VΔV = 12,0V. Полная энергия, отдаваемая аккумулятором мотоцикла, составляет ΔUцикл = (5000 ° C) (12,0 В) = (5000 ° C) (12,0 Дж / ° C) = 6,00 × 104 Дж. ΔUцикл = (5000 ° C) (12,0 В) = (5000 ° C) (12.0Дж / Кл) = 6,00 × 104Дж.

Аналогично для автомобильного аккумулятора q = 60,000 Cq = 60,000C и

ΔUcar = (60,000C) (12,0В) = 7,20 × 105Дж. ΔUcar = (60,000C) (12,0В) = 7,20 × 105Дж.
Значение
Напряжение и энергия связаны, но это не одно и то же. Напряжения батарей одинаковы, но энергия, подаваемая каждым из них, совершенно разная. Автомобильный аккумулятор требует запуска гораздо более мощного двигателя, чем мотоцикл. Также обратите внимание, что когда аккумулятор разряжен, часть его энергии используется внутри, а напряжение на его клеммах падает, например, когда фары тускнеют из-за разряда автомобильного аккумулятора.Энергия, подаваемая батареей, по-прежнему рассчитывается, как в этом примере, но не вся энергия доступна для внешнего использования.

Проверьте свое понимание 7.4

Сколько энергии имеет батарея AAA на 1,5 В, способная нагреться до 100 градусов Цельсия?

Обратите внимание, что энергии, вычисленные в предыдущем примере, являются абсолютными значениями. Изменение потенциальной энергии для аккумулятора отрицательное, так как он теряет энергию. Эти батареи, как и многие другие электрические системы, действительно перемещают отрицательный заряд — в частности, электроны.Батареи отталкивают электроны от своих отрицательных выводов ( A ) через любую задействованную схему и притягивают их к своим положительным выводам ( B ), как показано на рисунке 7.12. Изменение потенциала составляет ΔV = VB − VA = + 12 В ΔV = VB − VA = + 12 В, а заряд q отрицательный, так что ΔU = qΔVΔU = qΔV отрицательно, что означает, что потенциальная энергия батареи уменьшилась, когда q переместился с A на B .

Рисунок 7.12 Аккумулятор перемещает отрицательный заряд от отрицательной клеммы через фару к ее положительной клемме.Соответствующие комбинации химикатов в батарее разделяют заряды, так что отрицательный вывод имеет избыток отрицательного заряда, который отталкивается им и притягивается к избыточному положительному заряду на другом выводе. С точки зрения потенциала положительный вывод имеет более высокое напряжение, чем отрицательный. Внутри батареи движутся как положительные, так и отрицательные заряды.

Пример 7.5

Сколько электронов проходит через фару каждую секунду?
Когда автомобильный аккумулятор напряжением 12,0 В питает один аккумулятор 30.Фара 0-W, сколько электронов проходит через нее каждую секунду?
Стратегия
Чтобы узнать количество электронов, мы должны сначала найти заряд, который перемещается за 1,00 с. Перемещаемый заряд связан с напряжением и энергией посредством уравнений ΔU = qΔV.ΔU = qΔV. Лампа мощностью 30,0 Вт потребляет 30,0 джоулей в секунду. Поскольку батарея теряет энергию, мы имеем ΔU = -30JΔU = -30J, и, поскольку электроны переходят от отрицательной клеммы к положительной, мы видим, что ΔV = + 12.0V.ΔV = + 12.0V.
Решение
Чтобы найти заряд q перемещенного, решаем уравнение ΔU = qΔV: ΔU = qΔV:

Вводя значения ΔUΔU и ΔVΔV, получаем

q = -30.0J + 12,0V = -30,0J + 12,0J / C = -2,50C. Q = -30,0J + 12,0V = -30,0J + 12,0J / C = -2,50C.

Число электронов nene — это общий заряд, деленный на заряд одного электрона. То есть

ne = −2,50C − 1,60 · 10−19C / e− = 1,56 · 1019 электронов. ne = −2,50C − 1,60 · 10−19C / e− = 1,56 · 1019 электронов.
Значение
Это очень большое количество. Неудивительно, что мы обычно не наблюдаем отдельные электроны, так много которых присутствует в обычных системах. Фактически, электричество использовалось в течение многих десятилетий, прежде чем было установлено, что движущиеся заряды во многих обстоятельствах были отрицательными.Положительный заряд, движущийся в направлении, противоположном отрицательному, часто производит идентичные эффекты; это затрудняет определение того, что движется или оба движутся.

Проверьте свое понимание 7.5

Сколько электронов прошло через лампу мощностью 24,0 Вт?

Электрон-вольт

Энергия, приходящаяся на один электрон, очень мала в макроскопических ситуациях, подобных тому, что было в предыдущем примере — крошечная доля джоуля. Но в субмикроскопическом масштабе такая энергия, приходящаяся на одну частицу (электрон, протон или ион), может иметь большое значение.Например, даже крошечной доли джоуля может быть достаточно, чтобы эти частицы разрушили органические молекулы и повредили живые ткани. Частица может нанести ущерб при прямом столкновении или может создать опасные рентгеновские лучи, которые также могут нанести ущерб. Полезно иметь единицу энергии, относящуюся к субмикроскопическим эффектам.

На рис. 7.13 показана ситуация, связанная с определением такой единицы энергии. Электрон ускоряется между двумя заряженными металлическими пластинами, как это могло бы быть в телевизионной лампе или осциллографе старой модели.Электрон приобретает кинетическую энергию, которая позже преобразуется в другую форму — например, в свет в телевизионной трубке. (Обратите внимание, что с точки зрения энергии «спуск» для электрона означает «подъем» для положительного заряда.) Поскольку энергия связана с напряжением соотношением ΔU = qΔVΔU = qΔV, мы можем рассматривать джоуль как кулон-вольт.

Рис. 7.13 Типичная электронная пушка ускоряет электроны, используя разность потенциалов между двумя отдельными металлическими пластинами. По закону сохранения энергии кинетическая энергия должна равняться изменению потенциальной энергии, поэтому KE = qVKE = qV.Энергия электрона в электрон-вольтах численно равна напряжению между пластинами. Например, разность потенциалов 5000 В производит электроны с энергией 5000 эВ. Концептуальная конструкция, а именно две параллельные пластины с отверстием в одной, показана на (a), а реальная электронная пушка показана на (b).

Электрон-вольт

В субмикроскопическом масштабе удобнее определять единицу энергии, называемую электрон-вольт (эВ), которая представляет собой энергию, передаваемую фундаментальному заряду, ускоренному через разность потенциалов в 1 В.В форме уравнения,

1эВ = (1,60 × 10−19C) (1V) = (1,60 × 10−19C) (1J / C) = 1,60 × 10−19J. 1эВ = (1,60 × 10−19C) (1V) = (1,60 × 10− 19C) (1Дж / Кл) = 1,60 × 10−19Дж.

Электрону, ускоренному через разность потенциалов 1 В, придается энергия 1 эВ. Отсюда следует, что электрон, ускоренный до 50 В, приобретает 50 эВ. Разность потенциалов 100 000 В (100 кВ) дает электрону энергию 100 000 эВ (100 кэВ) и так далее. Точно так же ион с двойным положительным зарядом, ускоренный до 100 В, получает 200 эВ энергии.Эти простые соотношения между ускоряющим напряжением и зарядами частиц делают электрон-вольт простой и удобной единицей энергии в таких обстоятельствах.

Электрон-вольт обычно используется в субмикроскопических процессах — химическая валентная энергия, молекулярная и ядерная энергия связи входят в число величин, часто выражаемых в электрон-вольтах. Например, для разрушения некоторых органических молекул требуется около 5 эВ энергии. Если протон ускоряется из состояния покоя через разность потенциалов 30 кВ, он приобретает энергию 30 кэВ (30 000 эВ) и может разрушить до 6000 этих молекул (30 000 эВ ÷ 5 эВ на молекулу = 6000 молекул).(30,000 эВ ÷ 5 эВ на молекулу = 6000 молекул). Энергия ядерного распада составляет порядка 1 МэВ (1000000 эВ) на событие и, таким образом, может привести к значительному биологическому ущербу.

Сохранение энергии

Полная энергия системы сохраняется, если нет чистого добавления (или вычитания) из-за работы или теплопередачи. Для консервативных сил, таких как электростатическая сила, закон сохранения энергии утверждает, что механическая энергия постоянна.

Механическая энергия — это сумма кинетической энергии и потенциальной энергии системы; то есть K + U = константа.K + U = постоянная. Потеря U для заряженной частицы становится увеличением ее K . Сохранение энергии выражается в форме уравнения как

или

, где i и f обозначают начальные и конечные условия. Как мы уже много раз выясняли, учет энергии может дать нам понимание и облегчить решение проблем.

Пример 7.6

Электрическая потенциальная энергия, преобразованная в кинетическую энергию
Вычислите конечную скорость свободного электрона, ускоренного из состояния покоя через разность потенциалов 100 В.(Предположим, что это числовое значение имеет точность до трех значащих цифр.)
Стратегия
У нас есть система, в которой действуют только консервативные силы. Предполагая, что электрон ускоряется в вакууме, и пренебрегая гравитационной силой (мы проверим это предположение позже), вся электрическая потенциальная энергия преобразуется в кинетическую энергию. Мы можем идентифицировать начальную и конечную формы энергии как Ki = 0, Kf = 12mv2, Ui = qV, Uf = 0. Ki = 0, Kf = 12mv2, Ui = qV, Uf = 0.
Решение
Сохранение энергии утверждает, что

Вводя указанные выше формы, получаем

Решаем это для v :

Ввод значений для q , V и м дает

v = 2 (−1.60 × 10−19C) (- 100Дж / C) 9,11 × 10−31 кг = 5,93 × 106 м / зв = 2 (−1.60 × 10−19C) (- 100Дж / C) 9,11 × 10−31 кг = 5,93 × 106 м / с .
Значение
Обратите внимание, что и заряд, и начальное напряжение отрицательны, как показано на рисунке 7.13. Из обсуждения электрического заряда и электрического поля мы знаем, что электростатические силы, действующие на мелкие частицы, обычно очень велики по сравнению с силой тяжести. Большая конечная скорость подтверждает, что гравитационная сила здесь действительно незначительна. Большая скорость также указывает на то, насколько легко ускорить электроны с помощью малых напряжений из-за их очень малой массы.В электронных пушках обычно используются напряжения, намного превышающие 100 В. Эти более высокие напряжения вызывают настолько большие скорости электронов, что необходимо учитывать эффекты специальной теории относительности, и поэтому они оставлены для более поздней главы (Теория относительности). Вот почему в этом примере мы рассматриваем низкое напряжение (точно).

Проверьте свое понимание 7.6

Как этот пример изменится с позитроном? Позитрон идентичен электрону, за исключением того, что заряд положительный.

Напряжение и электрическое поле

До сих пор мы исследовали взаимосвязь между напряжением и энергией. Теперь мы хотим изучить взаимосвязь между напряжением и электрическим полем. Начнем с общего случая для неоднородного поля E → E →. Напомним, что наша общая формула для потенциальной энергии пробного заряда q в точке P относительно реперной точки R равна

UP = −RPF → · dl → .UP = −RPF → · dl →.

Когда мы подставляем в определение электрического поля (E → = F → / q), (E → = F → / q), это становится

UP = −q∫RPE → · dl →.UP = −q∫RPE → · dl →.

Применяя наше определение потенциала (V = U / q) (V = U / q) к этой потенциальной энергии, мы находим, что в общем случае

VP = −RPE → · dl → .VP = −RPE → · dl →.

7,6

Из нашего предыдущего обсуждения потенциальной энергии заряда в электрическом поле результат не зависит от выбранного пути, и, следовательно, мы можем выбрать наиболее удобный интегральный путь.

Рассмотрим частный случай положительного точечного заряда q в начале координат..доктор

, что упрощается до

Vr = −∫∞rkqr2dr = kqr − kq∞ = kqr.Vr = −∫∞rkqr2dr = kqr − kq∞ = kqr.

Этот результат,

— это стандартная форма потенциала точечного заряда. Это будет подробнее рассмотрено в следующем разделе.

Чтобы изучить еще один интересный частный случай, предположим, что однородное электрическое поле E → E → создается путем помещения разности потенциалов (или напряжения) ΔVΔV на две параллельные металлические пластины, обозначенные A и B (рис. 7.14). Изучение этой ситуации покажет нам, какое напряжение необходимо для создания определенной напряженности электрического поля.Это также покажет более фундаментальную взаимосвязь между электрическим потенциалом и электрическим полем.

Рисунок 7.14. Соотношение между V и E для параллельных проводящих пластин составляет E = V / dE = V / d. (Обратите внимание, что ΔV = VABΔV = VAB по величине. Для заряда, который перемещается от пластины A при более высоком потенциале к пластине B при более низком потенциале, необходимо включить знак минус следующим образом: −ΔV = VA − VB = VAB. − ΔV = VA − VB = VAB.)

С точки зрения физика, для описания любого взаимодействия между зарядами можно использовать ΔVΔV или E → E →.Однако ΔVΔV является скалярной величиной и не имеет направления, тогда как E → E → является векторной величиной, имеющей как величину, так и направление. (Обратите внимание, что величина электрического поля, скалярная величина, представлена ​​как E .) Связь между ΔVΔV и E → E → выявляется путем вычисления работы, совершаемой электрической силой при перемещении заряда из точки A к пункту B . Но, как отмечалось ранее, произвольное распределение заряда требует расчетов. Поэтому мы рассматриваем однородное электрическое поле как интересный частный случай.

Работа, совершаемая электрическим полем на рисунке 7.14 по перемещению положительного заряда q из A , положительная пластина, более высокий потенциал, в B , отрицательная пластина, более низкий потенциал, составляет

W = −ΔU = −qΔV.W = −ΔU = −qΔV.

Разница потенциалов между точками A и B составляет

−ΔV = — (VB − VA) = VA − VB = VAB. − ΔV = — (VB − VA) = VA − VB = VAB.

Если ввести это в выражение для работы, получится

Работа — это W = F → · d → = FdcosθW = F → · d → = Fdcosθ; здесь cosθ = 1cosθ = 1, так как путь параллелен полю.Таким образом, W = FdW = Fd. Поскольку F = qEF = qE, мы видим, что W = qEdW = qEd.

Подстановка этого выражения для работы в предыдущее уравнение дает

Заряд отменяется, поэтому для напряжения между точками A и B получаем

VAB = EdE = VABd} (только uniformE-field) VAB = EdE = VABd} (только uniformE-field)

, где d — это расстояние от A до B , или расстояние между пластинами на рисунке 7.14. Обратите внимание, что это уравнение подразумевает, что единицы измерения электрического поля — вольт на метр.Мы уже знаем, что единицы измерения электрического поля — ньютоны на кулон; таким образом, верно следующее соотношение между единицами:

Кроме того, мы можем продолжить это до интегральной формы. Подставляя уравнение 7.5 в наше определение разности потенциалов между точками A и B , мы получаем

VBA = VB − VA = −∫RBE → · dl → + ∫RAE → · dl → VBA = VB − VA = −∫RBE → · dl → + ∫RAE → · dl →

, что упрощается до

VB − VA = −ABE → · dl → .VB − VA = −∫ABE → · dl →.

В качестве демонстрации из этого мы можем вычислить разность потенциалов между двумя точками ( A и B ), равноудаленными от точечного заряда q в начале координат, как показано на рисунке 7.= 0 и, следовательно,

Этот результат, заключающийся в отсутствии разницы в потенциале вдоль постоянного радиуса от точечного заряда, пригодится при отображении потенциалов.

Пример 7.7

Какое максимально возможное напряжение между двумя пластинами?
Сухой воздух может поддерживать максимальную напряженность электрического поля около 3,0 × 106 В / м. 3,0 × 106 В / м. Выше этого значения поле создает достаточную ионизацию в воздухе, чтобы сделать воздух проводником. Это допускает разряд или искру, которые уменьшают поле.Каково же максимальное напряжение между двумя параллельными проводящими пластинами, разделенными 2,5 см сухого воздуха?
Стратегия
Нам дано максимальное электрическое поле E между пластинами и расстояние d между ними. Мы можем использовать уравнение VAB = EdVAB = Ed для расчета максимального напряжения.
Решение
Разность потенциалов или напряжение между пластинами составляет

Ввод заданных значений для E и d дает

VAB = (3,0 × 106 В / м) (0.025 м) = 7,5 × 104 VVAB = (3,0 × 106 В / м) (0,025 м) = 7,5 × 104 В

или

(Ответ состоит только из двух цифр, поскольку максимальная напряженность поля является приблизительной.)

Значение
Одним из следствий этого результата является то, что требуется около 75 кВ, чтобы совершить скачок искры через зазор размером 2,5 см (1 дюйм), или 150 кВ для искры 5 см. Это ограничивает напряжения, которые могут существовать между проводниками, возможно, на линии электропередачи. Меньшее напряжение может вызвать искру, если на поверхности есть шипы, поскольку острые точки имеют большую напряженность поля, чем гладкие поверхности.Влажный воздух разрушается при более низкой напряженности поля, а это означает, что меньшее напряжение заставит искру проскочить через влажный воздух. Наибольшие напряжения могут создаваться статическим электричеством в засушливые дни (рис. 7.16).

Рис. 7.16. Искровая камера используется для отслеживания траектории частиц высоких энергий. Ионизация, создаваемая частицами при прохождении через газ между пластинами, позволяет искре прыгнуть. Искры расположены перпендикулярно пластинам, следуя силовым линиям электрического поля между ними. Разность потенциалов между соседними пластинами недостаточно высока, чтобы вызвать искры без ионизации, производимой частицами из экспериментов с ускорителем (или космическими лучами).Эта форма детектора сейчас устарела и больше не используется, кроме как в демонстрационных целях. (кредит b: модификация работы Джека Коллинза)

Пример 7,8

Поле и сила внутри электронной пушки
Электронная пушка (рис. 7.13) имеет параллельные пластины, разделенные расстоянием 4,00 см, и дает электронам энергию 25,0 кэВ. а) Какова напряженность электрического поля между пластинами? (б) Какую силу это поле окажет на кусок пластика с зарядом 0,500 мкКл0,500 мкКл, который проходит между пластинами?
Стратегия
Поскольку напряжение и расстояние между пластинами указаны, напряженность электрического поля может быть рассчитана непосредственно из выражения E = VABdE = VABd.Зная напряженность электрического поля, мы можем найти силу, действующую на заряд, используя F → = qE → .F → = qE →. Поскольку электрическое поле имеет только одно направление, мы можем записать это уравнение в терминах величин: F = qEF = qE.
Решение
  1. Выражение для величины электрического поля между двумя однородными металлическими пластинами имеет вид Поскольку электрон является однозарядным и получает энергию 25,0 кэВ, разность потенциалов должна составлять 25,0 кВ. Ввод этого значения для VABVAB и разделения пластин 0.0400 м, получаем E = 25,0 кВ 0,0400 м = 6,25 × 105 В / м. E = 25,0 кВ 0,0400 м = 6,25 × 105 В / м.
  2. Величина силы, действующей на заряд в электрическом поле, получается из уравнения Подстановка известных значений дает F = (0,500 × 10–6C) (6,25 × 105 В / м) = 0,313 Н. F = (0,500 × 10–6 ° C) (6,25 × 105 В / м) = 0,313 Н.
Значение
Обратите внимание, что единицы измерения — ньютоны, так как 1V / m = 1N / C1V / m = 1N / C. Поскольку электрическое поле между пластинами однородно, сила, действующая на заряд, одинакова независимо от того, где находится заряд между пластинами.

Пример 7.9

Расчет потенциала точечного заряда
Учитывая точечный заряд q = + 2,0 нКл = + 2,0 нКл в начале координат, вычислите разность потенциалов между точкой P1P1 на расстоянии a = 4,0 см = 4,0 см от q и P2P2 на расстоянии b = 12,0 см = 12,0 см от q , где две точки имеют угол φ = 24 ° φ = 24 ° между собой (рисунок 7.17). Рисунок 7.17. Найдите разность потенциалов между P1P1 и P2P2.
Стратегия
Сделайте это в два этапа. Первый шаг — использовать VB − VA = −ABE → · dl → VB − VA = −ABE → · dl → и пусть A = a = 4.= 0 и, следовательно, ΔV = 0. ΔV = 0. Складывая две части вместе, получаем 300 В.

Значение
Мы продемонстрировали использование интегральной формы разности потенциалов для получения численного результата. Обратите внимание, что в этой конкретной системе мы могли бы также использовать формулу для потенциала из-за точечного заряда в двух точках и просто взять разницу.

Проверьте свое понимание 7.7

Из приведенных примеров, как энергия удара молнии зависит от высоты облаков над землей? Считайте систему облако-земля двумя параллельными пластинами.

Прежде чем описывать проблемы, связанные с электростатикой, мы предлагаем стратегию решения проблем, которой следует придерживаться для этой темы.

Стратегия решения проблем

Электростатика
  1. Изучите ситуацию, чтобы определить, присутствует ли статическое электричество; это может касаться отдельных стационарных зарядов, сил между ними и создаваемых ими электрических полей.
  2. Укажите интересующую систему. Это включает в себя указание количества, местоположения и типов взимаемых сборов.
  3. Определите, что именно необходимо определить в проблеме (определите неизвестные). Письменный список полезен. Определите, следует ли рассматривать кулоновскую силу напрямую — если да, может быть полезно нарисовать диаграмму свободного тела, используя силовые линии электрического поля.
  4. Составьте список того, что дано или может быть выведено из проблемы, как указано (укажите известные). Например, важно отличать кулоновскую силу F от электрического поля E .
  5. Решите соответствующее уравнение для определяемой величины (неизвестное значение) или проведите линии поля, как требуется.
  6. Изучите ответ, чтобы увидеть, разумен ли он: имеет ли он смысл? Правильные ли единицы и разумные ли числа?

Интегралы в электрических цепях

Производные и интегралы широко используются для описания переходных процессов в электрических цепях. Ниже мы рассмотрим некоторые типичные проблемы, которые можно решить с помощью интеграции.{{t_2}} {I \ left (t \ right) dt}, \]

, который представляет количество заряда, проходящего через провод между моментами времени \ (t = {t_1} \) и \ (t = {t_2}. \)

RC-схема

Простая последовательная RC-цепь — это электрическая цепь, состоящая из резистора и конденсатора.

Рисунок 1.

После того, как переключатель замкнут в момент времени \ (t = 0, \), ток начинает течь по цепи. Напряжение на резисторе определяется законом Ома:

.

\ [{V_R} \ left (t \ right) = I \ left (t \ right) R.{- \ frac {t} {{RC}}}}. \]

Рисунок 2.

Постоянная времени \ (\ tau = RC \) здесь определяет, насколько быстро происходит переходный процесс в цепи.

RL Схема

В простой цепи RL последовательно соединены резистор и катушка индуктивности.

Рисунок 3.

Когда переключатель в момент времени \ (t = 0 \) замкнут, применяется постоянная ЭДС \ (\ varepsilon \), и ток \ (I \) начинает течь по цепи.

Как и в предыдущем разделе, напряжение на резисторе равно

.

\ [{V_R} \ left (t \ right) = I \ left (t \ right) R.\]

Напряжение на катушке индуктивности выражается производной

\ [{V_L} \ left (t \ right) = L \ frac {{dI}} {{dt}}. {- \ frac {R} {L} t}}} \ right).\]

Рис. 4.

Мы видим, что постоянная времени для цепи RL определяется выражением \ (\ tau = \ large {\ frac {L} {R}} \ normalsize. \)

Мощность и энергия

Электрическая энергия \ (E, \), измеряемая в джоулях (Дж), представляет собой форму энергии, которая возникает из кинетической или потенциальной энергии, которой обладают электрические заряды.

Электрическая мощность \ (P, \), измеряемая в ваттах (Вт), — это скорость, с которой электрическая энергия передается по электрической цепи.

Мощность, рассеиваемая в элементе цепи постоянного тока \ (\ left ({DC} \ right) \), определяется формулой

\ [P = VI, \]

где \ (V \) — напряжение на элементе, а \ (I \) — ток в цепи.t {V \ left (s \ right) I \ left (s \ right) ds}, \]

где \ (s \) — внутренняя переменная интегрирования.

Энергия, накопленная в конденсаторе

Перемещение небольшого заряда \ (dq \) с одной пластины конденсатора на другую требует работы

\ [dW = Vdq = \ frac {q} {C} dq, \]

где \ (C \) — емкость, а \ (q \) — текущий заряд конденсатора. 2}}} {2}.2} — 4, & t \ gt 3 \ end {case}, \] где ток \ (I \) измеряется в \ (A \), а время \ (t \) измеряется в \ ({сек}. \). Найдите общий заряд, попавший в элемент за время \ (T = 6 \, с. \)

Пример 2

Ток в цепи увеличивается линейно во времени как \ (I \ left (t \ right) = \ alpha t \) в течение временного интервала \ (\ left [{0, T} \ right] \) и вызывает резистор \ (R \), чтобы нагреться. Предполагая, что процесс нагрева является адиабатическим, определите, как изменение температуры резистора \ (\ Delta T \) зависит от скорости \ (\ alpha.\) Удельная теплоемкость материала резистора \ (c, \), масса резистора \ (m. \)

Пример 3

Предположим, конденсатор \ (C \) заряжается от источника с постоянной ЭДС \ (\ varepsilon. \). Вычислите тепловую энергию, рассеиваемую резистором \ (R \) за время зарядки.

Пример 4

Когда переключатель замкнут в момент времени \ (t = 0, \), начальный ток в цепи без источника \ (RL \) равен \ ({I_0} = 1 \, A. \) Найдите энергию \ ({E_R } \) рассеивается резистором между \ (t = 0 \) и \ (T = 1 \, ms, \), если \ (R = 50 \, k \ Omega, \) \ (L = 0.{- 100t}} \, \ left (V \ right).} \] Определите полную энергию, рассеиваемую элементом между \ (t = 0 \) и \ (t = 10 \, {ms}. \)

Пример 6

В момент времени \ (t = 0, \) ЭДС \ (\ varepsilon = 50 \, V \) применяется к первоначально незаряженному конденсатору \ (C = 10 \, \ mu F. \). Конденсатор начинает заряжаться через резистор \ (R = 100 \, k \ Omega. \) Определите количество электронов на отрицательной пластине конденсатора за \ (1 \) секунду.

Пример 7

Ток и напряжение на элементе схемы изменяются по синусоидальному закону: \ [{I \ left (t \ right) = {I_0} \ sin \ left ({\ frac {{2 \ pi t}} {T} + \ theta} \ right), \; \;} \ kern0pt { V \ left (t \ right) = {V_0} \ sin \ left ({\ frac {{2 \ pi t}} {T}} \ right),} \] где \ (T \) — период колебаний, \ (\ theta \) — разность фаз, \ ({I_0} \) и \ ({V_0} \) — начальные значения тока и напряжения. 2} — 4, & t \ gt 3 \ end {case}, \] где ток \ (I \) измеряется в \ (A \), а время \ (t \) измеряется в \ ({сек}.6} = {9 + \ left ({\ frac {{216}} {3} — 4} \ right) — \ left ({3 — 12} \ right)} = {60 \, C}. \]

Пример 2.

Ток в цепи увеличивается линейно во времени как \ (I \ left (t \ right) = \ alpha t \) в течение временного интервала \ (\ left [{0, T} \ right] \) и вызывает резистор \ (R \), чтобы нагреться. Предполагая, что процесс нагрева является адиабатическим, определите, как изменение температуры резистора \ (\ Delta T \) зависит от скорости \ (\ alpha. \). Удельная теплоемкость материала резистора равна \ (c, \) масса резистора \ (м.2}. \]

Таким образом, изменение температуры \ (\ Delta \ theta \) пропорционально квадрату текущей скорости \ (\ alpha \).

Пример 3.

Предположим, конденсатор \ (C \) заряжается от источника с постоянной ЭДС \ (\ varepsilon. \). Вычислите тепловую энергию, рассеиваемую резистором \ (R \) за время зарядки. {- \ frac {t } {{RC}}}}.{15}}} \]

Пример 7.

Ток и напряжение на элементе схемы изменяются по синусоидальному закону: \ [{I \ left (t \ right) = {I_0} \ sin \ left ({\ frac {{2 \ pi t}} {T} + \ theta} \ right), \; \;} \ kern0pt { V \ left (t \ right) = {V_0} \ sin \ left ({\ frac {{2 \ pi t}} {T}} \ right),} \] где \ (T \) — период колебаний, \ (\ theta \) — разность фаз, \ ({I_0} \) и \ ({V_0} \) — начальные значения тока и напряжения. Найдите среднюю мощность, рассеиваемую в элементе схемы за период одного цикла.T {\ sin \ left ({\ frac {{2 \ pi t}} {T} + \ theta} \ right) \ sin \ left ({\ frac {{2 \ pi t}} {T}} \ right ) dt}.} \]

Использование идентичности произведения на сумму

\ [{\ sin \ alpha \ sin \ beta \ text {=}} \ kern0pt {\ frac {1} {2} \ left [{\ cos \ left ({\ alpha — \ beta} \ right) — \ cos \ left ({\ alpha + \ beta} \ right)} \ right],} \]

подынтегральное выражение можно переписать в виде

\ [{\ sin \ left ({\ frac {{2 \ pi t}} {T} + \ theta} \ right) \ sin \ left ({\ frac {{2 \ pi t}} {T}}) \ right)} = {\ frac {1} {2} \ left [{\ cos \ left ({- \ theta} \ right) — \ cos \ left ({\ frac {{4 \ pi t}}} {T } + \ theta} \ right)} \ right]} = {\ frac {1} {2} \ left [{\ cos \ theta — \ cos \ left ({\ frac {{4 \ pi t}} {T } + \ theta} \ right)} \ right].T} = {\ frac {{{I_0} {V_0}}} {2} \ left [{\ cos \ theta — \ underbrace {\ frac {{\ sin \ left ({4 \ pi + \ theta} \ right ) — \ sin \ theta}} {{4 \ pi}}} _ 0} \ right]} = {\ frac {{{I_0} {V_0} \ cos \ theta}} {2}.} \]

Как видите, максимальная средняя мощность достигается при \ (\ theta = 0: \)

\ [{{\ bar P} _ {\ max}} = \ frac {{{I_0} {V_0}}} {2}. \]

Пример 8.

Источник постоянной ЭДС \ (\ varepsilon = 100 \, V \) подключается к цепи с начальным сопротивлением \ ({R_0} = 20 \, \ Omega.\) Рассчитайте заряд \ (Q \), который будет проходить в цепи в течение \ (T = 1 \, min, \), если сопротивление линейно увеличивается со скоростью \ (\ alpha = 1 \ large {\ frac {\ Омега} {s}} \ normalsize. \)

Решение.

Сопротивление \ (R \) цепи изменяется по закону

\ [R \ left (t \ right) = {R_0} + \ alpha t. \]

По закону Ома,

\ [I \ left (t \ right) = \ frac {\ varepsilon} {{R \ left (t \ right)}} = \ frac {\ varepsilon} {{{R_0} + \ alpha t}}. \ ]

Чтобы найти заряд \ (Q, \), мы интегрируем ток \ (I \ left (t \ right) \) за интервал времени \ (\ left [{0, T} \ right], \), где \ ( T = 1 \, min = 60 \, с.T} = {\ frac {\ varepsilon} {\ alpha} \ left [{\ ln \ left ({{R_0} + \ alpha T} \ right) — \ ln {R_0}} \ right]} = {\ frac {\ varepsilon} {\ alpha} \ ln \ frac {{{R_0} + \ alpha T}} {{{R_0}}}} = {\ frac {\ varepsilon} {\ alpha} \ ln \ left ({1 + \ frac {{\ alpha T}} {{{R_0}}}} \ right).} \]

Подставляя указанные значения, получаем

\ [{Q = \ frac {\ varepsilon} {\ alpha} \ ln \ left ({1 + \ frac {{\ alpha T}} {{{R_0}}}} \ right)} = {\ frac { {100}} {1} \ ln \ left ({1 + \ frac {{1 \ times 60}} {{20}}} \ right)} \ приблизительно {138.6 \, C} \]

.
Разное

Добавить комментарий

Ваш адрес email не будет опубликован. Обязательные поля помечены *